You are on page 1of 57

1. A 6-year-old African American boy presents to the emergency department compla ining of abdominal pain.

He has a history of sickle cell disease and experience d an episode of sickle cell crisis six months ago. His medical history is other wise unremarkable. Physical examination shows a thin boy in apparent distress. He is clutching his abdomen, whimpering, and lying uncomfortably on the examina tion table. His vital signs include a temperature of 38C (100.4F), blood pressu re of 110/70 mm Hg, pulse of 92/min, and respirations of 24/min. His tympanic m embranes are pearly gray and the light reflex is present. No exudates or lesion s are visible in the oropharynx. Chest auscultation is unremarkable and heart s ounds are normal. He does not allow you to palpate his abdomen. What is the next best step in the management of this patient? A. Intravenous fluids B. Ibuprofen PO for pain relief C. Ketorolac IV for pain relief D. Urinalysis and 24-hour measurement of urinary output E. Peripheral blood smear to evaluate for sickled cells Answer: A

The sickle cell crisis is an episode of acute pain that can range in severity fr om minimal to agonizing. It is the most common type of vasocclusive event in pa tients with sickle cell disease. Although most of these crises have no identifi able cause, events such as dehydration, cold, infection, stress, menses, alcohol consumption, and nocturnal hypoxemia are known triggers. Some studies have als o shown hemoglobin >8.5 g/dL to be a major risk factor for the development of an acute painful episode. Various areas of the body can be affected, with the mos t common locations including the back, chest, extremities, and abdomen. Objecti ve clinical signs such as fever or tachypnea are seen approximately half the tim e. The first step in management of patients (especially children) presenting wi th an acute painful episode is the administration of fluids orally or intravenou sly to ensure optimal hydration (Choice A). Ibuprofen (Choice B) is not typically administered to patients in sickle cell cr isis, as it does not provide adequate pain relief. Intravenous morphine is the most commonly prescribed analgesic in sickle cell pa tients. Another option used (especially in patients stricken with bone pain) is the non-steroidal anti-inflammatory drug ketorolac (Choice C), which provides p ain relief superior to meperidine. However, fluid resuscitation should be perfo rmed first. Urinalysis and the measurement of urinary output (Choice D) would be particularl y helpful if urinary tract infection or dehydration were suspected in a patient with sickle cell crisis. However, ordering these tests would not be the very fi rst step in management of sickle cell crisis. In a patient with sickle cell disease, the peripheral blood smear (Choice E) usu ally reveals irreversible sickling of 5-50% of red blood cells. However, the pr esence of sickled cells is not diagnostic of an acute pain episode. Unfortunate ly, there are no laboratory tests that can be used to differentiate this type of crisis from baseline condition. Educational Objective: The first step in the management of patients (especially children) presenting wi th sickle cell crisis is the intravenous administration of fluids. 64% of people answered this question correctly.

2. A 6-year-old African American boy presents to the emergency department compla ining of abdominal pain. He has a history of sickle cell disease and experience d an episode of sickle cell crisis six months ago. His medical history is other wise unremarkable. Physical examination shows a thin boy in apparent distress. He is clutching his abdomen, whimpering, and lying uncomfortably on the examina tion table. His vital signs include a temperature of 38C (100.4F), blood pressu re of 110/70 mm Hg, pulse of 92/min, and respirations of 24/min. His tympanic m embranes are pearly gray and the light reflex is present. No exudates or lesion s are visible in the oropharynx. Chest auscultation is unremarkable and heart s ounds are normal. He does not allow you to palpate his abdomen The first step in the management of this acute pain crisis is undertaken. The c hild now allows for a brief physical examination of his abdomen, which is signif icant for splenomegaly. What is the next best step in management of this patien t? A. Blood and urine cultures B. Chest x-ray C. Complete blood count D. Intravenous antibiotics E. Warn the boy?s parents that he should avoid contact sports until the splenom egaly resolves Answer: C

Young children with sickle cell disease have a 30% risk of developing splenic se questration crisis (SCC) and are more susceptible because their spleens have not yet become fibrotic. In SCC, a dramatic fall in hemoglobin concentration occur s secondary to vaso-occlusion within the spleen and splenic pooling of red blood cells. Typically the spleen rapidly enlarges, and the patient can go on to exp erience hypovolemic shock. Patients with SCC who do not receive blood transfusi ons in time have a mortality rate of 10-15%. Therefore, a complete blood count (Choice C) should be ordered next for this child with splenomegaly. Blood and urine cultures (Choice A) are appropriate inclusions in the workup of febrile sickle cell patients and should ideally be obtained before antibiotics a re administered. However, splenic sequestration crisis is of greater concern in this patient. Chest x-ray (Choice B) is an appropriate inclusion in the workup of febrile sick le cell patients, especially those with signs or symptoms of pulmonary involveme nt. However, splenic sequestration crisis is of greater concern in this patient . Hospitalization and administration of intravenous antibiotics (Choice D) is reco mmended for those sickle cell disease patients in pain who are febrile (>40.0C), appear toxic, or are not receiving prophylactic penicillin. Patients with splenomegaly are at increased risk for splenic rupture, either fro m trauma or infiltrative disease that breaks the splenic capsule. Although the rupture itself may be painless, intraperitoneal bleeding, shock, and death can r esult. Therefore this child would be well advised to avoid contact sports until the splenomegaly resolves (Choice E). Since this warning is not immediately re levant, however, it can be delayed until discharge. Educational Objective: A complete blood count should be obtained immediately in sickle cell disease pat ients who present with abdominal pain and splenomegaly, as this patient group is at risk for developing splenic sequestration crisis. 48% of people answered this question correctly.

3. A healthy 23-year-old Caucasian woman comes to the physician for a routine he alth maintenance examination. She has no complaints. She has no medical proble ms. She does not use tobacco, alcohol, or drugs. Her family history is not sig nificant. She takes no medication. Her vital signs are within normal limits. Examination shows no abnormalities. Her Pap smear, performed three days ago, re veals "atypical squamous cells of unknown significance" (ASCUS). Which of the f ollowing is the most appropriate next step in the management of this patient? A. B. C. D. E. Refer for colposcopy Obtain HPV testing on the sample Treat with doxycycline and repeat Pap smear in 6 weeks Schedule repeat Pap smear in 3 months Reassurance and repeat Pap smear in one year B

Answer:

There are different approaches for the evaluation of ASCUS. Among the above cho ices, HPV DNA testing is the most important next step in management. In this me thod, samples are collected both for cytology and HPV DNA, and if cytology resul ts are negative, the sample for HPV DNA is discarded. If cytology results are p ositive, HPV DNA testing is performed, and if this test is positive for high-ris k HPV type, an immediate colposcopy is performed. If the test is negative for h igh-risk HPV type, the Pap smear is repeated after one year. (Choice A) An immediate colposcopy may be done to evaluate ASCUS, but this will result in a high percentage of false positive cases. (Choices C and D) Obtaining accelerated (every four to six months) serial Pap sm ears is another approach for the evaluation of ASCUS. In this method, serial Pa p smear testing is done until two consecutive Pap smears are normal. If a secon d Pap smear is still abnormal, a colposcopy is performed. This approach is less favored because of the greater number of follow-up visits and delays in the dia gnosis. (Choice E) Patients with ASCUS on cervical cytology possess a high risk of an un derlying high-grade dysplasia; therefore, further diagnostic testing is recommen ded. Educational Objective: HPV DNA testing is the best way to evaluate atypical squamous cells of unknown s ignificance. 22% of people answered this question correctly.

4. A 78-year-old white male who lives in a nursing home was sent to the physicia n?s office because he is having recurrent falls. The nursing home staff noticed that he has a progressive decrease in vision. The patient denies double vision , dizziness, nausea, vomiting, headache, or vertigo. His other medical problems include Alzheimer?s dementia, peptic ulcer disease, diverticulosis, and degener ative joint disease. He has been taking donepezil, omeprazole, acetaminophen, a nd multivitamin tablets. His vital signs are stable. On examination, he has de creased vision in both eyes, and bilateral anterior cortical cataracts. Which o

f the following associated conditions is most likely contributing to his decreas ed vision? A. B. C. D. E. Open-angle glaucoma Closed-angle glaucoma Proliferative retinopathy Macular degeneration Retinal detachment D

Answer:

The common causes of decreased vision in elderly patients (especially above 75 y ears of age) are cortical cataracts and associated macular degeneration. The pr evalence of cataracts in this age group is almost 40%, and that of associated ma cular degeneration is 20%. These patients should be thoroughly evaluated for th e severity of macular degeneration, as they may not benefit from cataract surger y. (Choices A and B) Even though the prevalence rates of glaucoma increase with age , it is still less than 10%. (Choice C) Proliferative retinopathy is usually seen in diabetic patients. Witho ut the history of diabetes, this diagnosis is very unlikely. (Choice E) Retinal detachment usually presents with a sudden onset of unilateral loss of vision. Educational Objective: Cataracts and associated macular degeneration are extremely common in the elderl y population and often result in visual impairment. Elderly patients should be thoroughly evaluated for the severity of macular degeneration, as they may not b enefit from cataract surgery.

5. A 27-year-old gravida 2, para 1 Caucasian woman at 38 weeks of gestation was admitted to the labor and delivery unit after the spontaneous rupture of membran es. After an unremarkable labor she vaginally delivered a son weighing 3720 gra ms (8 lbs, 3 oz). Shortly after birth, the infant was found to have hemolytic d isease of the newborn secondary to Rh incompatibility. The infant?s serum total bilirubin concentrations then rose rapidly and now, at 24 hours after delivery, are measured at 22.8 mg/dL. Intensive phototherapy has not adequately reduced the bilirubin concentration in the infant?s blood. The parents of this child a re Jehovah?s Witness adherents and refuse the proposed exchange transfusion. Wit hout the transfusion, the infant will likely suffer severe brain damage or death . What is the next best step in managing this boy?s care? A. Administer RhoGAM to mother and infant B. Continue intensive phototherapy for additional 72 hours C. Proceed immediately with exchange transfusion to prevent kernicterus D. Consult with the hospital?s ethics committee about seeking court injunction to mandate exchange transfusion E. Transfer infant to another hospital Answer: D

In difficult ethical situations such as this one, medical practitioners must see k to balance the autonomy of the family with the welfare of the child. Parental wishes should be honored within certain parameters. For instance, when surgeri es are performed on Jehovah?s Witness adherents (be they adults or children), sp ecific blood-sparing techniques or erythropoietin may allow for excellent outcom es. However, in circumstances in which an infant?s survival is at stake, it is appropriate for the medical team to seek approval from the courts to proceed wit h a blood transfusion (Choice D). Continuing the intensive phototherapy (Choice B) is inadvisable. Typically, pho totherapy will result in a decrease in the bilirubin within 4-6 hours. Waiting another 72 hours puts the child at serious risk for developing kernicterus. In a R (D)-negative woman carrying a fetus that is or may be R (D)-positive, Rho GAM (anti-D immune globulin) must be given as a prophylactic measure at 28 weeks of gestation, after clinical procedures or trauma that can cause fetomaternal h emorrhage, and at the conclusion of any pregnancy. Once alloimmunization to the Rh (D) antigen has been established, however, RhoGAM administration (Choice A) is no longer effective. It therefore should not be administered to the mother i n this case. Moreover, there is no current indication for the administration of RhoGAM to children, including those with hemolytic disease of the newborn. Proceeding with the exchange transfusion (Choice C) may save the infant?s life, but invites litigation if done without first obtaining the approval of the court . Transferring the infant to another hospital (Choice E) irresponsibly evades the ethical conflict by placing the child?s care in the hands of physicians who do n ot know his history. Transfers should only be initiated if the hospital current ly caring for the boy is unable to properly address his medical issues, which wa s not implied in this case. Educational Objective: In providing medical care, clinicians must seek to balance the autonomy of the f amily with the welfare of the child. If the child?s life is endangered, it is a ppropriate for the clinician to seek approval from the courts to proceed with tr eatments deemed medically necessary. 23% of people answered this question correctly.

6. A 55-year-old Caucasian female presents to the emergency department with comp laints of severe back pain and difficulty walking. She describes the back pain at the mid-thoracic level, which wraps around her upper abdomen in a band-like f ashion bilaterally. She also complains of weakness, numbness, and tingling in b oth her lower extremities. Her symptoms started approximately two weeks ago and are progressively getting worse. She denies any difficulty with bowel or bladd er function. She has a past history of non-Hodgkin?s lymphoma, which was approp riately treated eight months ago with chemotherapy and radiation therapy. Her n eurological examination reveals a motor strength of 3/5 in both lower extremitie s. There is hyperreflexia of the knee and ankle reflexes, and a positive Babins ki?s sign bilaterally. Which of the following is the most appropriate next step in the management of this patient? A. B. C. D. Give low-dose corticosteroids and consult radiation oncologist Give high-dose corticosteroids and order MRI of the spine Emergent surgical decompression with posterior laminectomy Get MRI of the spine and call the surgeon for emergent surgical decompressio

n E. Call the radiation oncologist for therapy Answer: E

Epidural spinal cord compression (ESCC) is a common complication of cancers, and it is sometimes the initial presentation of cancer. It frequently causes pain a nd some degree of neurological dysfunction. A metastatic tumor from any primary site can cause ESCC. Some of the common cancers with a tendency to metastasize t o the spinal column are: prostrate, breast and lung cancer, non-Hodgkin?s lympho ma, and renal cell carcinoma. This patient presents with a classic thoracic radicular pain (wraps around the a bdomen) and neurological symptoms of ESCC. It is a medical emergency, and the tr eatment should be instituted as soon as possible. High-dose corticosteroids (esp ecially dexamethasone) should be administered immediately, and MRI of the spine should be obtained to confirm the diagnosis. Corticosteroids exert their effect by decreasing the edema and swelling around the tumor tissue, thereby decreasing the compression. Radiation therapy is the definitive treatment of choice for most patients. The b est response is seen in patients with radiosensitive tumors such as lymphoma, mu ltiple myeloma, breast, and prostrate cancer. Once the MRI confirms the diagnosi s, radiation therapy should be started. (Choices A and B) High-dose corticosteroids are considered a part of standard th erapy for ESCC. These should always be used along with radiation therapy. (Choices C and D) Lymphoma is extremely radiosensitive and should be treated wit h radiation therapy. Moreover, in studies, posterior decompression with a lamine ctomy with or without radiotherapy has not been shown to be superior to radiothe rapy alone. Radical surgical resection with tumor debulking has shown to improve outcomes if done early and should be considered in patients with ESCC, due to t umors other than lymphoma. Educational Objective: Symptomatic epidural spinal cord compression should be treated emergently with a combination of high-dose corticosteroids and radiation therapy. MRI is the inve stigative procedure of choice. 68% of people answered this question correctly.

7. A 36-year-old Caucasian woman comes to the physician for a routine health mai ntenance examination. She is requesting a prescription for oral contraceptive p ills because she has a new boyfriend. Her other medical problems include hypert ension, severe migraine headaches, bronchial asthma and gastroesophageal reflux disease. She does not use tobacco, alcohol, or drugs. Her mother has ovarian c ancer. Her father has diabetes mellitus. Her maternal grandmother was diagnose d with ovarian cancer at the age of 56 years, and her paternal grandfather was d iagnosed with prostate cancer at the age of 70 years. Her medications include h ydrochlorothiazide, albuterol metered dose inhalers, and famotidine. Her blood pressure is 120/80mm Hg and pulse is 76/min. Examination shows no abnormalities . The presence of which of the following makes this patient an unsuitable candi date for oral contraceptive pills? A. Her age and nonsmoking history B. Her history of bronchial asthma

C. Her well controlled hypertension D. Strong family history of ovarian cancer E. Her history of migraine headaches Answer: E

The absolute contraindications to the use of oral contraceptives are the followi ng: history of thromboembolic event or stroke active liver disease history of estrogen dependent tumor pregnancy abnormal uterine bleeding heavy smokers who are older than 35 hypertriglyceridemia Relative contraindications are the following: migraine headaches poorly controlled hypertension anticonvulsant drug therapy (Choice F) Diabetes mellitus, bronchial asthma, and a family history of malignan cies are not contraindications to the use of oral contraceptives. Educational Objective: Migraine headache is a relative contraindication to the use of oral contraceptiv e pills. 61% of people answered this question correctly.

8. A 26-year-old Caucasian female comes to your office because of fever, sweatin g, heat intolerance, and shortness of breath. She also has pain in front of her neck. Her temperature is 38.3 C (101 F), blood pressure is 156/60 mmHg, pulse is 110/min, and respirations are 22/min. The thyroid gland is slightly enlarged an d tender. Extremities are warm and moist. Other systems are essentially unremark able. Thyroid function tests are as follows: T3: T4: TSH: The is A. B. C. D. E. 256 ng/dL (normal 80-180 ng/dL) 15 mg/dL (normal 4-12 mg/dL) < 0.01 mU/mL (normal 0.35 ? 5.0 mU/mL) erythrocyte sedimentation rate is 90 mm/hr. A radioactive iodine uptake scan mostly likely to reveal: Decreased uptake Increased diffuse uptake Increased patchy uptake Hot nodule Normal uptake A

Answer:

This patient has classical clinical manifestations of subacute thyroiditis, whic h is characterized by very low radioiodine uptake (generally less than 5% at 24 hours). Thyrotoxicosis in subacute thyroiditis is due to the release of stored t hyroid hormones by inflammatory damage. Viral infections have been implicated as a cause of subacute thyroiditis. Thyrotoxicosis with low radioactive iodine upt ake is also seen in painless thyroiditis, postpartum thyroiditis, surreptitious thyroid hormone administration, and iodine-induced thyroiditis.

(Choices B, C, and D) Increased diffuse uptake is seen in toxic multinodular goi ter, and increased patchy uptake is seen in toxic adenoma. All these diseases ha ve an overproduction of thyroid hormones. Radioactive iodine uptake is generally increased; however, this may normalize after treatment. Educational Objective: Thyrotoxicosis with low radioactive iodine uptake is seen in subacute thyroiditi s.

9. A 26-year-old Caucasian female comes to your office because of fever, sweatin g, heat intolerance, and shortness of breath. She also has pain in front of her neck. Her temperature is 38.3 C (101 F), blood pressure is 156/60 mmHg, pulse is 110/min, and respirations are 22/min. The thyroid gland is slightly enlarged an d tender. Extremities are warm and moist. Other systems are essentially unremark able. Thyroid function tests are as follows: T3: T4: TSH: 256 ng/dL (normal 80-180 ng/dL) 15 mg/dL (normal 4-12 mg/dL) < 0.01 mU/mL (normal 0.35 ? 5.0 mU/mL)

What is the best treatment option for this patient? A. Propylthiouracil B. Methimazole C. Radioactive Iodine D. Systemic antibiotics E. NSAID and beta-blocker Answer: E

As explained before, thyrotoxicosis in patients with subacute thyroiditis is due to the release of pre-formed thyroid hormones; therefore, antithyroid drugs (Ch oices A and B), which work by decreasing the synthesis of thyroid hormones, are ineffective. Due to the same reason, radioactive iodine (Choice C) is incorrect. Standard therapy for subacute thyroiditis consists of a NSAID usually combined with beta-blockers (Choice E). Rarely, a systemic glucocorticoid (like prednison e) is required for short period in patients who are very sick and do not respond to standard therapy. The thyrotoxic phase lasts for a few weeks and is then usu ally followed by a phase of hypothyroidism that can last for a few months. Treat ment with thyroxine may be temporarily required during the hypothyroid phase in severely symptomatic patients. The majority of patients with subacute thyroiditi s become euthyroid. (Choice D) Occasionally, suppurative infection of the thyroid gland is difficult to differentiate from subacute thyroiditis. Patients with bacterial infections of the thyroid gland are usually not thyrotoxic because the involvemement in the thyroid gland is central. In suppurative thyroiditis, the left thyroid lobe is commonly involved (85%) and skin overlying the thyroid gland is erythematous. Th yroid ultrasound may reveal an abscess formation. Bacterial thyroid infections a re treated with systemic antibiotics, and surgical drainage is required in more than 60% of the patients.

Educational Objective: Standard therapy for subacute thyroiditis consists of a NSAID usually combined w ith beta-blockers. 62% of people answered this question correctly. 10. A 32-year-old gravida 3, para 1 Caucasian woman at 38 weeks of gestation is admitted to the labor and delivery unit after complaining of uterine contraction s spaced ten minutes apart. Her medical records are not currently available, bu t the patient states she had a normal pregnancy without any complications. Duri ng her initial pelvic examination, several flesh-colored, hyperkeratotic papules are discovered on her posterior introitus, labia major, and labia minora. The papules range from 1-2 centimeters in diameter, have a smooth surface, and are s essile. The patient denies any awareness of the lesions and says she has no his tory of sexually transmitted diseases. Her cervix is 70% effaced and dilated to 3 centimeters. What is the next best step in managing this situation? A. B. C. D. E. Allow labor to continue without any intervention Administer Pitocin to accelerate labor Administer magnesium sulfate for tocolysis Administer acyclovir intravenously Perform cesarean section A

Answer:

Condyloma acuminata is the dermatologic manifestation of an infection with the h uman papilloma virus, with over 90% of such condylomas arising from HPV subtypes 6 and 11 specifically. HPV is primarily transmitted through sexual contact, an d the areas affected include the penis, vulva, vagina, cervix, perineum, and per ianal region. Less frequently, HPV may be found in the oropharynx, larynx, or t rachea secondary to oral-genital contact or secondary to vertical transmission f rom mother to infant during childbirth. One relatively common benign laryngeal tumor in children, recurrent respiratory papillomatosis, is caused by the acquis ition of HPV during passage through the vaginal canal. However, since HPV is th ought to be contracted by the infant in less than 1% of all childbirths to women who have condylomas, no intervention is usually undertaken (Choice A). Pitocin (Choice B) is not indicated since the woman was just admitted and her la bor is progressing satisfactorily at this time. Tocolysis with magnesium sulfate (Choice C) is not necessary as this woman is at 38 weeks of gestation and is expected to have a normal vaginal delivery. Acyclovir (Choice D) is a popular and effective medication for the treatment of herpes simplex virus (HSV), not human papilloma virus (HPV). Topical medications such as trichloroacetic acid, podophyllin, and fluorouracil are used in treatin g HPV. Cesarean section (Choice E) was often done in previous decades as a preventive m easure to avoid HPV transmission. It is now only done for HPV infection if the condylomas are thought to be so large that they might bleed excessively during a vaginal childbirth. Educational Objective: Condyloma acuminata is a manifestation of infection with HPV and is not consider ed a contraindication to vaginal delivery in the pregnant woman. 44% of people answered this question correctly.

11. A 41-year-old Hispanic woman comes to a family practice clinic to discuss he r desire to stop smoking. Her husband was just diagnosed with lung cancer, and she is now extremely concerned about her own risk of developing lung cancer. Sh e has smoked 1/2 pack of cigarettes daily for the past 20 years. On several occ asions, she attempted to stop smoking by using the nicotine patch or nicotine gu m, but always relapsed within the first week. She is now inquiring about other available options. Which of the following medications is commonly prescribed fo r smoking cessation? A. B. C. D. E. Fluoxetine Venlafaxine Trazodone Bupropion Citalopram D

Answer:

Because nicotine is so highly addictive, smoking cessation is a difficult undert aking for many patients. Some individuals find nicotine replacement therapy (e. g., nicotine gum, nicotine patch, nicotine inhaler) helpful in the quest to stop smoking. Another typically well-tolerated option is sustained release bupropio n, an antidepressant that inhibits neuronal uptake of norepinephrine, serotonin, and dopamine. The combination of bupropion and nicotine replacement therapy is particularly effective. (Choices A and E) Fluoxetine and citalopram are antidepressants categorized as s elective serotonin reuptake inhibitors. They are not typically prescribed for s moking cessation. (Choice B) Venlafaxine is an antidepressant that inhibits norepinephrine, seroto nin, and dopamine reuptake. It is not typically prescribed for smoking cessation . (Choice C) Trazodone is an antidepressant that inhibits serotonin reuptake. It is not typically prescribed for smoking cessation. Educational Objective: Bupropion is a drug prescribed both as an antidepressant and for smoking cessati on. 95% of people answered this question correctly.

12. A 57-year-old Caucasian female presents to the primary care physician for a follow-up visit after undergoing a routine colonoscopy. The colonoscopy reveale d adenocarcinoma of the ascending colon and two well-differentiated sessile aden omatous polyps of the sigmoid colon. Past medical history is significant for mi tral valve prolapse and fibromyalgia. Past surgical history is significant for hysterectomy. Medications include hormone replacement therapy and ibuprofen. T he patient eats a high-fiber vegetarian diet, consumes 3-4 alcoholic beverages p er day, and has a remote 10-pack-year history of smoking tobacco. She rarely ex ercises. Family history is negative for cancer. The patient is very concerned about being diagnosed with colon cancer. Which of the following factors placed her at greatest risk for developing the disease?

A. B. C. D. E.

Alcohol intake NSAID intake Tobacco use Hormone replacement therapy High fiber diet A

Answer:

Several studies have suggested a strong correlation between alcohol intake (Choi ce A) and the development of colon cancer, especially in those individuals consu ming 45 grams or more of alcohol per day (for sake of comparison, the amount of alcohol in a 12 oz beer is 13 grams). It is thought that the mechanism responsi ble may involve interference of folate absorption or decreased folate intake. Regular NSAID intake (Choice B), hormone replacement therapy in postmenopausal w omen (Choice D), and a diet high in fruits in vegetables (Choice E) are all thou ght to offer some protection against the development of colon cancer. Tobacco use (Choice C) has been linked to an increase in both incidence and mort ality of colon cancer. However, this woman?s remote 10-pack-year history of smok ing is likely not the most significant factor in the development of her colon ca ncer, especially given her heavy alcohol consumption. Educational Objective: Colon cancer has been associated with significant alcohol intake and cigarette s moking. Fibrous diets rich in fruit and vegetables, regular NSAID use, hormone replacement therapy, and regular exercise have been identified as protective fac tors. 22% of people answered this question correctly.

13. A 22-year-old primigravida is hospitalized at 34 weeks gestation because of blurred vision, headache, and pain in the right upper quadrant of the abdomen. Her temperature is 36.7 C (98 F), blood pressure is 220/110 mmHg, pulse is 80/mi n, and respirations are 20/min. The fundoscopic exam is normal. On examination , there is swelling of her face and both her hands, bilateral exaggeration of de ep tendon reflexes with clonus, and a positive Babinski. The pelvic exam shows 50% effacement, and 3 cm dilation of the cervix. While you are obtaining IV acc ess, the patient starts to have generalized tonic-clonic seizures. An airway is secured, and breathing is present. Urinalysis reveals proteinuria of 3+. Which of the following is the most effective strategy to decrease this patient's risk for developing further complications? A. Check vital signs every four hours B. Start magnesium sulfate infusion C. Speed vaginal delivery D. Start parenteral clonidine E. Start phenytoin infusion Answer: C

This patient was admitted to the hospital because she has a severe preeclampsia, which was later complicated with eclampsia. Patients with severe preeclampsia are at greater risk of developing eclampsia. The first priority in patients wit h eclampsia or postictal coma is respiratory and cardiovascular resuscitation.

Anticonvulsant medications can be administered after placing two large-bore need les in the patient. Although the most effective agent for hemodynamic and seizu re control is magnesium sulfate, the most effective treatment to prevent further complications is to accelerate delivery. Eclampsia can cause several other com plications besides seizures, such as disseminated intravascular coagulopathy, ac ute renal failure, hepatocellular injury, liver rupture, intracerebral hemorrhag e, etc. Magnesium sulfate prevents seizures, but it will not stop the pathologi c process. (Choice B) Although magnesium sulfate is beneficial, it is not as effective as p regnancy termination. This would have been a correct choice if the question had asked about the best next step in the management of this patient, because hemod ynamic stability and seizure control are important to attain before attempting d elivery. (Choice D) Clonidine is not indicated in this setting. Either hydralazine or la betalol are the antihypertensive drugs of choice. (Choice E) Phenytoin or diazepam is not as effective as magnesium sulfate in con trolling seizures. Furthermore, as mentioned above, seizures are not the only c omplication of eclampsia. Speeding up the delivery is the most important. (Choice A) Frequent monitoring of vital signs is part of the management, but it will not prevent the patient from developing further complications. Educational Objective: Eclampsia is a serious complication of pregnancy. If the patient is in the thir d trimester, especially in the last six weeks, termination of pregnancy is advis ed in order to stop the pathologic process. There is no pharmacologic therapy m ore effective than this intervention. 40% of people answered this question correctly.

14. A 22-year-old primigravida is hospitalized at 34 weeks gestation because of blurred vision, headache, and pain in the right upper quadrant of the abdomen. Her temperature is 36.7 C (98 F), blood pressure is 220/110 mmHg, pulse is 80/mi n, and respirations are 20/min. The fundoscopic exam is normal. On examination , there is swelling of her face and both her hands, bilateral exaggeration of de ep tendon reflexes with clonus, and a positive Babinski. The pelvic exam shows 50% effacement, and 3 cm dilation of the cervix. While you are obtaining IV acc ess, the patient starts to have generalized tonic-clonic seizures. An airway is secured, and breathing is present. Urinalysis reveals proteinuria of 3+. Which of the following is the most effective agent to treat this patient's hyper tension? A. Methyldopa B. Labetalol C. Enalapril D. Amlodipine E. Atenolol Answer: B

In pregnant patients with a hypertensive crisis, either hydralazine or labetalol are the antihypertensive drugs of choice.

(Choice C) ACE inhibitors are contraindicated in pregnancy. (Choice A) Methyldopa is the preferred agent for oral therapy in mild to moderat e hypertension. (Choice D) Calcium channel blockers are added to methyldopa as second line agent s. (Choice E) Atenolol is an oral agent (in USA) and is not indicated in the acute setting. Although beta-blockers are considered to be safe, there are some repor ts of impaired fetal growth, especially with atenolol if used in the early part of a pregnancy. Educational Objective: Either hydralazine or labetalol are the drugs of choice for the acute management of hypertension during pregnancy. Methyldopa is preferred for oral therapy in mild to moderate hypertension during pregnancy.

15. A 22-year-old primigravida is hospitalized at 34 weeks gestation because of blurred vision, headache, and pain in the right upper quadrant of the abdomen. Her temperature is 36.7 C (98 F), blood pressure is 220/110 mmHg, pulse is 80/mi n, and respirations are 20/min. The fundoscopic exam is normal. On examination , there is swelling of her face and both her hands, bilateral exaggeration of de ep tendon reflexes with clonus, and a positive Babinski. The pelvic exam shows 50% effacement, and 3 cm dilation of the cervix. While you are obtaining IV acc ess, the patient starts to have generalized tonic-clonic seizures. An airway is secured, and breathing is present. Urinalysis reveals proteinuria of 3+. During labor, the patient has another seizure. Which of the following is the mo st appropriate pharmacotherapy in order to avoid seizure recurrence in this pati ent? A. Phenytoin B. Magnesium sulfate C. Phenobarbital D. Diazepam E. Valproic acid Answer: B

Anti-seizure prophylaxis in a patient with eclampsia has been a topic of prolong ed debate. Recently, some studies have confirmed that magnesium sulfate is not only the best anticonvulsant medication for patients with eclampsia, but it is a lso the more effective agent to prevent further seizures. (Choice D) With diazepam, there is a greater risk of respiratory depression. Ma gnesium sulfate has been proven to be more effective and to have a low neonatal morbidity. Diazepam is more useful in the setting of status epilepticus, or if the patient has contraindications to use magnesium sulfate (e.g., myasthenia gra vis). (Choice A) Phenytoin is also useful, but it is not as effective as magnesium sul fate.

(Choice C) Phenobarbital is reserved only for those cases in which seizures pers ist, despite the use of magnesium sulfate, diazepam, or phenytoin. (Choice E) Valproic acid is not part of the therapy of eclampsia. Educational Objective: The best medication to prevent further seizures in a patient with eclampsia is m agnesium sulfate. Diazepam or phenytoin can be added to the therapy if seizures persist, although the use of diazepam should be limited due to its depressant e ffects on the fetus. 73% of people answered this question correctly. 16. A 22-year-old primigravida is hospitalized at 34 weeks gestation because of blurred vision, headache, and pain in the right upper quadrant of the abdomen. Her temperature is 36.7 C (98 F), blood pressure is 220/110 mmHg, pulse is 80/mi n, and respirations are 20/min. The fundoscopic exam is normal. On examination , there is swelling of her face and both her hands, bilateral exaggeration of de ep tendon reflexes with clonus, and a positive Babinski. The pelvic exam shows 50% effacement, and 3 cm dilation of the cervix. While you are obtaining IV acc ess, the patient starts to have generalized tonic-clonic seizures. An airway is secured, and breathing is present. Urinalysis reveals proteinuria of 3+. Which of the following is considered an extremely ominous sign/feature of this c ondition? A. Increased PGI 2 and Thromboxane A 2 ratio B. Retinal hemorrhages C. Glomerular capillary endotheliosis D. Microangiopathic hemolytic anemia E. Subcapsular hematoma of the liver Answer: B

This patient has preeclampsia complicated by eclampsia. Retinal hemorrhage is c onsidered to be an extremely ominous sign, because it reflects the vascular dama ge that has occurred in other organs. Retinal vasospasm can also be seen in pre eclampsia, and can be visualized on ophthalmoscopic examination. (Choice A) Opposite to normal pregnancy, the PGI 2 to Thromboxane A 2 ratio decr eases, and does not increase. This change results in an increase in peripheral resistance, thereby leading to the clinical symptoms and complications of preecl ampsia and eclampsia. A deficiency in nitric oxide, as well as an increase in E ndothelin-I, have also been incriminated -- the former being a vasodilator and t he latter a potent vasoconstrictor. (Choice C) Glomerular capillary endotheliosis is the typical glomerular lesion o f preeclampsia/eclampsia. It involves a marked swelling of the glomerular capil lary endothelium and deposits of fibrinoid material in and beneath the endotheli al cells. On light microscopy, the glomerular diameter is increased, with endot helial and mesangial cell swelling. (Choice D) Microangiopathic hemolytic anemia can occur in preeclampsia and eclam psia, but it is not considered an ominous sign. It results from the injury of R BC by the damaged endothelium that is usually associated with the condition. (Choice E) Vasoconstriction of the hepatic vasculature can result in necrosis an d hemorrhage of the periportal spaces and ultimately, subcapsular hematoma. Educational Objective:

Retinal hemorrhage is an extremely ominous sign of preeclampsia/eclampsia. 17. A 16-month-old African American boy is brought to the emergency department ( ED) by his mother with several hours history of unlocalized pain and crying. Hi s past medical history is insignificant, except for a runny nose that his mother noticed yesterday. He has been developing normally and has met all the develop mental milestones appropriately. All his immunizations are up-to-date. His blo od pressure is 100/70 mmHg, pulse is 130/min, temperature is 36.7C (98F) and res pirations are 30/min. His physical examination is significant for peripheral cy anosis and symmetric swelling of the hands and feet. Which of the following wou ld be most helpful in establishing the cause of this patient?s problem? A. B. C. D. E. Hemoglobin electrophoresis Chest x-ray Blood cultures Digital systolic pressure Aspirin trial A

Answer:

The classical clinical scenario of a vaso-occlusive painful crisis of sickle cel l disease is described. This presents in infants as painful swelling of the han ds and feet (dactylitis or hand-foot syndrome), and can be the first manifestati on of sickle cell disease. Most bony vaso-occlusive events occur primarily with in the bone marrow cavity. Most are multifocal and associated with mild tendern ess and localized edema. In older children and adults, these episodes occur pri marily in the joints, back and chest. Hemoglobin electrophoresis is employed to establish the diagnosis. (Choices B and C) Chest x-ray and blood cultures are frequently used as part of the work-up in patients with painful crises to reveal the precipitating factor. (Choice D) Digital systolic pressure in response to cooling is occasionally used to confirm the diagnosis of Raynaud phenomenon. Educational Objective: Vaso-occlusive painful crisis presents in infants as painful swelling of the han ds and feet (dactylitis or hand-foot syndrome); this can be the first manifestat ion of sickle cell disease.

18. A 52-year-old Caucasian woman calls you at 1:00 a.m. on a Saturday due to se vere, low back pain. She is a patient of your physician friend who is away for the weekend. She is very upset and angry because she had to call multiple times and was put on hold before she finally got in touch with you. She tells you th at she has had low back pain for many years, and her physician usually prescribe s oxycodone for the pain. Her pain is much worse, and she is unable to sleep at all. She wants you to prescribe something stronger, so that she can sleep thro ugh the night. You do not have any records available with you at this time, but she does not appear to be drug-seeking, based on the history. What is the most appropriate reaction to the patient's request? A. Ask her politely to calm down and call you back in the morning.

B. C. D. E.

Ask the patient to take an extra dose of her pain medication. Inquire about her pain in detail. Ask her to call her primary care physician on Monday. Give her long-acting morphine for pain relief. C

Answer:

Chronic, low back pain is a common problem encountered in the primary care setti ng. It is not unusual to see a lot of patients being treated with long-term nar cotic analgesics. The primary goal and challenge in the management of such pati ents is to recognize and treat the pain adequately and appropriately, and to avo id drug abuse and dependency at the same time. It is difficult to recognize the pattern of drug use or abuse in patients who have an underlying chronic disorde r that causes pain. It becomes even difficult in the absence of a long-term rel ationship with the patient (as in the above vignette). It is important, however , to recognize any change in the pain character or severity, which should not be assumed to be due to underlying medical condition and should be thoroughly inve stigated. The patient in the above vignette should be inquired about her curren t symptoms in detail and should be managed accordingly. (Choices A and D) Asking the patient to call back while she is having acute pain is unethical and inappropriate. (Choices B and E) The patient should be inquired regarding her pain complaints and triage based on her response. Her worsening pain should not be automaticall y assumed to be from her chronic back pain. Increasing the dose of her current medications or prescribing a more potent narcotic analgesic is not appropriate w ithout further investigation into the cause of her worsening symptoms. Educational Objective: Any change in the character or intensity of pain in a patient with chronic pain syndrome should be thoroughly investigated. 87% of people answered this question correctly. 19. A 50-year-old African-American man comes to the emergency department (ED) be cause his left leg has been bothering him a lot. He has a vague and generalized pain in his left leg, and this is present both at rest and with activity. Ther e is also marked swelling of the left leg as compared to the right lower extremi ty. He was recently discharged from the hospital one month ago ago after being diagnosed and treated for deep venous thrombosis of the left leg. Since his dis charge from the hospital, he has been taking 2.5 mg of warfarin daily in the mor ning. His INR (International Normalized Ratio) in the ED is 1.4. A venogram re veals recurrent acute venous thrombosis extending into the internal iliac vein a nd inferior vena cava. He does not have any other medical illnesses and is not taking any medications. At this point, which of the following would you recomme nd for this patient? A. B. C. D. E. Increase the dose of warfarin to bring INR within therapeutic range Switch him to Coumadin Interrupt the inferior vena cava with a filter Ask him to take the warfarin dose at bedtime Discontinue warfarin and begin thrombolytic therapy A

Answer:

This patient has recurrent deep venous thrombosis with a more proximal extension into the inferior vena cava. This is most likely due to inadequate anticoagula tion with warfarin. The anticoagulant effect of warfarin is mediated by the inh

ibition of vitamin K dependent gamma-carboxylation of factors II, VII, IX, and X . Although long-term therapy with warfarin is highly effective and causes a sig nificant reduction in the frequency of recurrent venous thromboembolism, the dos age required to attain a therapeutic INR (International Normalized Ratio) is ind ividualized for each patient. The dosage of warfarin is mainly determined by the difference in the genetically determined rate of drug metabolism and the patient?s vitamin K status. Warfari n is metabolized in the liver by the hepatic cytochrome P450 2C9 isoenzyme, whic h exhibits multiple genetic polymorphisms that are responsible for the individua lized dose requirements of patients. Furthermore, patients on a vitamin K-rich diet (green leafy vegetables) can have difficulty attaining a therapeutic INR; t hus, they require higher doses of warfarin to achieve the desired INR. In this case, the patient's dosage of warfarin should be increased , and he shou ld be followed closely with serial measurements of PT/INR. (Choice B) Warfarin is the generic form of Coumadin. Switching from warfarin to a different agent or brand name is not indicated at this point. (Choice C) Insertion of an IVC filter is usually indicated only in patients with acute venous thromboembolism who have a contraindication to anticoagulant thera py or who continue to have recurrent venous thromboembolism despite adequate ant icoagulation. (Choice D) Changing the dose schedule does not affect the bioavailability of the drug and is therefore not required. (Choice E) The use of thrombolytic therapy in the treatment of acute deep venous thrombosis is controversial and is associated with a higher complication rate. It is only considered if there is significant swelling which may compromise the arterial circulation to the extremity. Educational Objective: The dosage required to attain a therapeutic INR (International Normalized Ratio) is individualized for each patient. Insertion of an IVC filter is usually indi cated only in patients with acute venous thromboembolism who have a contraindica tion to anticoagulant therapy or who continue to have recurrent venous thromboem bolism despite adequate anticoagulation. 43% of people answered this question correctly. 20. A 17-year-old Caucasian male is brought to the emergency department by his f riend due to acute shortness of breath. He is apparently in acute distress and speaks in broken sentences. His friend says that they were playing soccer and h e went to the bushes to retrieve the ball. Soon after, he returned to the field and complained of shortness of breath and itching. The patient does not recall any bite or injury. The physical examination reveals widespread wheals all ove r the body. What is the best next step in the management of this patient? A. Complete physical examination looking for a tick B. Epinephrine subcutaneously C. Epinephrine IV followed by IV fluids D. IV hydrocortisone and fluids E. IV diphenhydramine and furosemide Answer: C

The clinical scenario described is consistent with an anaphylactic reaction. Th e two most typical causes of death in patients with anaphylactic reaction are re spiratory failure and cardiovascular collapse. Respiratory failure usually resu lts from airway obstruction due to bronchospasm and/or laryngeal edema; other co mplications that lead to respiratory failure are cardiogenic or noncardiogenic p ulmonary edema, or acute respiratory distress syndrome. Cardiovascular collapse results from increased vascular permeability and hypovolemia, alterations in pe ripheral vascular resistance, and myocardial depression. Due to the potentially life-threatening nature of anaphylaxis, prompt treatment is required. Epinephr ine is the drug of choice for anaphylaxis because it can reverse associated hypo tension and bronchospasm. This patient seems to be in acute distress (broken sp eech indicates high degree of respiratory compromise) and should be given epinep hrine IV. In addition, fluid support should also be given to compensate for the relative hypovolemia due to the increased vascular permeability. (Choice A) Epinephrine should be given immediately because fatality rates are hi ghest in patients whose treatment with epinephrine is delayed. (Choice B) Subcutaneous epinephrine can be used in patients without significant respiratory or cardiovascular symptoms. (Choices E and D) Antihistamine drugs and corticosteroids are usually given to p atients with anaphylactic reactions, but these are less important in reversing a cute life-threatening complications than epinephrine. Educational Objective: Epinephrine is the drug of choice for anaphylaxis because it can reverse the ass ociated hypotension and bronchospasm. It should be given immediately because fa tality rates are highest in patients whose treatment with epinephrine is delayed . 38% of people answered this question correctly. 21. A 17-year-old Caucasian male is brought to the emergency department by his f riend due to acute shortness of breath. He is apparently in acute distress and speaks in broken sentences. His friend says that they were playing soccer and h e went to the bushes to retrieve the ball. Soon after, he returned to the field and complained of shortness of breath and itching. The patient does not recall any bite or injury. The physical examination reveals widespread wheals all ove r the body. The patient is appropriately treated. He states that he has never had such symp toms before, which scared him, but he wants to go home now. The most recent phy sical examination reveals minimal rash and clear lungs on auscultation. What is the most important step in the management of this patient? A. Immediate discharge B. Advise to wear a sign that indicates his allergy C. Prescribe a daily antihistamine drug D. Instruct how to use EpiPen injector E. Advice not to play on that playground Answer: D

The most important intervention from the list is instructing the patient about t he proper use of EpiPen. EpiPen is an epinephrine automatic injector which shou ld always be immediately available in case of recurrence. Such a device may be lifesaving if patients are inadvertently re-exposed to the offending agent. The EpiPen for adults delivers 0.3 mL of 1:1000 epinephrine (0.3 mg). The injector is administered by taking off the cap and pushing the opposite end firmly into the upper lateral thigh. The needle is delivered into the thigh automatically.

The patient should count to 10 before removing the pen to insure complete deliv ery of the medication. The patient should also be instructed to obtain immediat e medical care after the injection of the drug. (Choice B) Another important measure would be wearing a Medic Alert bracelet or similar device at all times. This information can expedite diagnosis and approp riate treatment. (Choice A) Because of the potential for biphasic reactions, the patient should b e observed for as long as 24 hours in severe episodes of anaphylaxis. (Choice E) The patients should be instructed to avoid the exposure if the allerg en that caused the reaction is identified. (Choice C) Daily antihistamines are prescribed to patients with frequent anaphyl actic reactions if the allergen can not be determined or avoided. Educational Objective: The most important intervention following an episode of anaphylaxis is instructi ng the patient about the proper use of EpiPen. 75% of people answered this question correctly. 22. A healthy 24-year-old Caucasian woman comes to the physician for a routine h ealth maintenance examination. She says her sister was recently diagnosed with systemic lupus erythematosus and has had several complications. She is requesti ng you to do a blood test to diagnose it early. She has no complaints. She has no other medical problems. She does not drink alcohol. She has smoked one pac k of cigarettes daily for five years. Her mother has colon cancer. Her vital s igns are within normal limits. Examination shows no abnormalities. You order a serum antinuclear antibody test, and it came back positive at 1:80 titers (Norm al is less than 1:40). Other labs are unremarkable. Which of the following is the most appropriate course of action? A. B. C. D. E. Order anti-dsDNA antibody levels. Order anti-Smith and anti-dsDNA antibody levels. Order serum complement and ESR. Explain to her that she does not require further testing. Start her on low-dose hydroxychloroquine. D

Answer:

ANA tests are highly sensitive (>95%) for SLE and are positive in almost all pat ients with SLE; however positive ANA test results are commonly found in around 2 0-25% of the normal population. A study conducted on 125 normal individuals fou nd an ANA titer above 1:40 in 32 percent, above 1:80 in 13 percent, and above 1: 320 in three percent. In the absence of any SLE symptoms, elevated ANA in low t iters has no significance. If the ANA test result is negative and the clinical picture is not highly suggestive of SLE, no further workup for SLE is indicated. The reason that many physicians still order this test, despite knowing the fact that it can yield false positive results, is that they can reassure the patient if the test is negative; however, if the test becomes positive, many will end up having further rheumatologic workup. In the absence of SLE symptoms, this test should therefore not be ordered. Educational Objective: ANA tests are highly sensitive (>95%) for SLE and are positive in almost all pat

ients with SLE; however, positive ANA test results are commonly found in around 20-25% of the normal population. In the absence of any SLE symptoms, an elevate d ANA titer has no significance. 23. A mother brings in her 3-month-old son to the office because she is concerne d about his "spitting up." The infant?s vomiting has increased in the past mont h, and occurs primarily at night. The vomit is typically composed of curdled fo rmula, and is not bilious or projectile. The infant had a decrease in appetite, but his weight has been stable. His mother reports that the infant is mildly i rritable after feedings, but does not appear to experience significant pain. Ba sed on this presentation, functional gastroesophageal reflux is considered the m ost likely diagnosis. What should you recommend to the mother as the next best step? A. B. C. D. E. Prone positioning at night Thicken formula with cereal Prescription of ranitidine to protect esophageal lining Prescription of cisapride as prokinetic therapy Surgical repair with Nissen fundoplication B

Answer:

Gastroesophageal reflux (GER) is a clinical diagnosis. Reassurance should be of fered to the mother that the "spitting up" is a normal occurrence in infants up to 24 months old. It typically requires no intervention if the child is otherw ise healthy and developing appropriately (the "happy spitter"). In children wit h mild GER symptoms, the mainstay of conservative treatment is thickening of for mula with cereal, which usually results in decreased emesis, decreased cry, and better weight gain. (Choice A) Prone positioning is another conservative treatment that may alleviat e symptoms; however, this treatment method is of some concern, because of the co rrelation between prone positioning and SIDS. Formula thickening should be atte mpted first. (Choice C) H2 receptor antagonists such as ranitidine are appropriate in those i nfants with a more severe GER presentation and who have failed conservative trea tment. (Choice D) Cisapride enhances myenteric plexus acetylcholine release, and is hig hly effective in increasing gastrointestinal motility; however, because cisaprid e can cause cardiac arrhythmias in some individuals, its availability is heavily restricted in the United States. (Choice E) Surgery is reserved for cases of GER that do not respond to medical m anagement. Educational Objective: Functional GER is extremely common in infants, and should be initially addressed with reassurance and formula thickening. Prescription medication and surgery a re reserved for more severe cases of GER which have failed conservative treatmen t. 24. A 70-year-old Caucasian male presents to the emergency department due to rig ht-sided arm and leg weakness that occurred several hours ago. The episode last ed 15 minutes and resolved spontaneously. He had a similar episode of right arm weakness two days ago. He was diagnosed with hypercholesterolemia on routine c heck-up three months ago, and has been treated with pravastatin. He does not sm oke or consume alcohol. His blood pressure is 140/90 mmHg and his heart rate is 70/min. Doppler studies shows 89% stenosis in his left carotid artery and 40% stenosis in his right carotid artery. He undergoes left carotid endarterectomy. The surgery goes well and he recovers from the anesthesia without any complica

tions. In the recovery room, upon protrusion of the tongue, the tongue deviates to the left. Which of the following structures is most likely damaged? A. B. C. D. E. Left vagus nerve Left hypoglossal nerve Left recurrent laryngeal nerve Right facial nerve Ansa hypoglossus nerve B

Answer:

A number of nerve injuries can occur during carotid endarterectomy. Inadvertent retraction or transection of the hypoglossal nerve causes tongue deviation to t he site of injury, as observed in this patient. (Choice C) Recurrent laryngeal nerve injury leads to unilateral vocal cord paral ysis, with a resultant change in voice quality. This nerve lies distal to the a rea of carotid dissection . (Choice D) The facial nerve can be damaged after it exits from the stylomastoid foramen and courses along the inferior portion of the ear. Its marginal mandibu lar branch (which supplies the orbicularis oris muscle) is most commonly injured , with a resultant asymmetric smile. (Choice A) The vagus nerve lies posterolaterally in the carotid sheath and is al so at risk for injury during the procedure. (Choice E) The ansa hypoglossus nerve innervates the strap muscles of the neck; unlike the other nerves, this nerve may be sacrificed with impunity. Educational Objective: A number of nerve injuries can occur during carotid endarterectomy. Inadvertent retraction or transection of the hypoglossal nerve causes tongue deviation to t he site of injury. *Extremely important question for USMLE step-3 25. A 50-year-old female was found to have abnormal thyroid function on routine blood testing. Her TSH level is 9 mU/ml (0.35 to 5.0 mU/ml is normal) and free T 4 is 1.3 ng/dl (normal 0.8 to 1.8 ng/dl). She is currently asymptomatic and is t aking no medications. She denies use of tobacco, alcohol, and drugs. Family hist ory is positive for a mother with hypothyroidism. She has four siblings, all are alive and well. Her menstrual cycles are regular. Examination is unremarkable. Her CBC, serum chemistries, and lipid profile are within normal limits. Which of the following will be most useful in guiding her therapy? A. B. C. D. E. T3 measurement Antithyroid peroxidase (anti-TPO) measurement Thyroid ultrasound Radioactive iodine uptake and scan Thyroglobulin measurement B

Answer:

The patient has subclinical hypothyroidism, which is defined as a mild elevation in TSH levels (5 to 10) accompanied by normal free T4 levels. Treatment is warr anted in the presence of (1) antithyroid antibodies, (2) an abnormal lipid prof ile, (3) symptoms of hypothyroidism, and (4) ovulatory and menstrual dysfunctio n. When antithyroid antibodies are present with elevated TSH, there is a high ch

ance for a patient to become overtly hypothyroid. A patient with a TSH level > 1 0 mU/ml is also generally treated with levothyroxine. However, the downside of t he treatment in asymptomatic patients is the risk of overtreatment, leading to i ncreased bone loss and atrial fibrillation. Routine screening of thyroid functio ns is controversial. The American Thyroid Association (ATA) recommends all indiv iduals over the age of 40 to be screened for thyroid dysfunction. The American C ollege of Physicians recommends screening women over 50 years with findings sugg estive of thyroid disease. (Choices A, D, and E) T3 measurement, radioactive iodine uptake, and thyroglobul in measurement play no role in the management of subclinical hypothyroidism. (Choice C) Thyroid ultrasound could be useful if thyroid enlargement is present, in order to monitor any change in size and look for dominant nodules (> 1 cm in diameter). The patient's physical exam is normal; therefore, thyroid ultrasound in the above case will not help in making management decisions. Educational Objective: Asymptomatic subclinical hypothyroidism does not require treatment. Treatment is warranted in the presence of: - antithyroid antibodies - an abnormal lipid profile - symptoms of hypothyroidism - ovulatory and menstrual dysfunction 29% of people answered this question correctly. 26. A 50-year-old Caucasian male presents to your office for routine check-up. He says, ?I gained some weight recently, and I know I should give up fast food.? He complains of frequent heartburns that is relieved with food intake and over -the-counter antacids. His past medical history is insignificant. He does not smoke currently, but he used to smoke 1-2 packs of cigarettes daily for 25 years . He consumes 1-2 bottles of beer on weekends. His blood pressure is 150/90 mm Hg and heart rate is 85/min. His occult fecal blood test is positive. You reco mmend esophagogastroscopy, but he replies that the procedure would give him a lo t of discomfort. He has heard of ?video pills? that he can swallow and ?these w ould show the stomach.? He requests for such a pill. What is the best response to this patient?s request? A. B. C. D. E. No such pills exist. You must be misinformed Video pills have low resolution and are therefore inferior to endoscopy Video pills have limited view and endoscopy is clearly preferred Video pills may be associated with high rates of GI perforation Video pills are a good alternative to endoscopy in your case E

Answer:

The patient is most probably talking about wireless video endoscopy, a novel tec hnique that is gaining popularity in gastroenterology. A video capsule is taken by the patient and video recording is traced by the outside sensors. Currently , it has been proven to be an effective tool to diagnose some small bowel diseas e because visualization of the majority of the small bowel mucosa is not possibl e with push endoscopy. It can be helpful to identify the source of small bowel hemorrhage, tumors, ulceration and inflammatory conditions. (Choice B) The images acquired are of excellent resolution and have an 8:1 magni fication, which is higher than that of conventional endoscopes. (Choice E) Limited views of the esophagus, stomach and cecum are obtained by thi

s technique; therefore, in no way can it currently replace endoscopy for the dia gnosis of esophageal and stomach disease. (Choice D) Complications of wireless video endoscopy are rare. Retention of the capsule happens in less than 1% of patients. Educational Objective: Wireless video endoscopy is an effective tool to diagnose some small bowel disea se because visualization of the majority of the small bowel mucosa is not possib le with push endoscopy. Endoscopy is preferred for the diagnosis of esophageal and stomach disease. 27. A 77-year-old Caucasian man is brought to the office by his daughter because of hematuria. He complains of gross, painless hematuria for the past week. He denies frequency, urgency, hesitancy, or dribbling. His other medical problems include hypertension, chronic low back pain, and chronic renal insufficiency. He has smoked one pack of cigarettes daily for 46 years. His medications includ e an acetaminophen and codeine combination, amlodipine, and hydrochlorothiazide. Rectal examination shows no abnormalities. The patient?s labs reveal the foll owing: Urine: Specific gravity: 1.009 Blood: Gross Leukocyte esterase: Negative Nitrites: Negative WBC: 1-2/hpf RBC: many/hpf Serum Chemistry: Serum Na: 141 mEq/L Serum K: 4.3 mEq/L Chloride: 105 mEq/L Bicarbonate: 20 mEq/L BUN: 53 mg/dL Serum Creatinine: 2.5 mg/dL Ultrasonogram of the kidneys shows bilateral cortical atrophy but no other lesio ns. Which of the following is the most appropriate next step in the management o f this patient? A. B. C. D. E. Cystoscopy Intravenous pyelography Prostate-specific antigen Urine cytology Abdominal CT scan A

Answer:

This patient has a high risk of urinary tract malignancy, especially bladder or prostate cancer. This possibility has to be kept in mind in any elderly patient with painless hematuria. Exposure to aniline dyes and a significant smoking hi story increase the patient's risk. Cystoscopy is the gold standard in the detec tion of bladder malignancy, and it is also a very useful test to detect prostate cancer. USG is used if there is any suspected kidney mass. (Choice E) An abdominal CT scan would have been the first option if the patient was young, and if kidney stones were being sought. Since he has renal insuffici ency, giving contrast for CT scan may worsen his condition.

(Choice B) IVP is not advisable because the patient has renal failure. To look for ureteral tumors, a retrograde pyelogram at the time of cystoscopy is an alte rnative approach in these patients. (Choice C) An elevated PSA level will suggest the presence of prostate malignanc y, but it will not detect bladder carcinoma. The likelihood of bladder carcinom a in this patient is higher than that of prostate cancer. (Choice D) Urine cytology has a lower diagnostic yield than cystoscopy; however, it can sometimes identify bladder malignancy when cystoscopy is negative. This is why it is often performed during cystoscopy. Urine cytology alone has a fal se negative rate of 65-80%. If the cytology is positive, and cystoscopy is nega tive, evaluation for upper tract and the prostatic urethra tumors should be soug ht . Educational Objective: The presence of hematuria should always be a matter of concern, especially if an elderly patient presents with painless hematuria. IVP is superior in the detec tion of malignancies of the upper urinary tract; however, its use is contraindic ated in renal failure. Cystoscopy can reveal malignancies of the bladder and pr ostate. Kidney cancers are best evaluated with USG or CT scan. 28. A 35-year-old man comes to the emergency department after being involved in a snowmobile accident. He complains of severe pain in his right groin and is un able to extend his right knee. He has no other past medical problems. He is an avid athlete and runs five miles every day. On physical examination, he is una ble to extend his right knee against resistance; passive movements are within no rmal limits. There is a sensory loss over the anterior and medial thigh, medial shin, and the arch of the right foot. The knee jerk is markedly decreased on t he right side. Which of the following nerve injuries can most likely explain th e physical findings? A. B. C. D. E. Repeat Pap in 6 months Sciatic nerve injury Obturator nerve injury Peroneal nerve injury Prepare for extubation E

Answer:

This patient is most likely suffering from a traumatic femoral nerve injury, whi ch is an uncommon lower extremity nerve injury due to the nerve's location withi n the pelvis and anterior thigh. On examination, patients with significant femo ral neuropathies exhibit weakness involving the quadriceps muscle group, with sp aring of leg adduction (which is a function of the obturator nerve). This muscl e weakness is exemplified by an inability to extend the knee against resistance. In addition, sensory loss over the anterior aspect and most of the medial aspe ct of the thigh is typical. This sensory loss extends down the medial shin to t he arch of the foot due to dysfunction of the saphenous nerve (branch of the fem oral nerve). The knee jerk also decreases in amplitude or becomes absent. (Choice B) A patient with a significant sciatic nerve injury will complain of we akness affecting most of the lower leg musculature, including the hamstrings. H ip flexion, extension, abduction, adduction, and knee extension are usually norm al. There is sensory loss involving the lower leg. In contrast to a femoral ne rve injury, the medial calf and arch of the foot may be spared secondary to the preserved innervation by the saphenous nerve (a branch of the femoral nerve). F urthemore, sensation is spared above the knee, both anteriorly and posteriorly. The knee jerk is normal, but the ankle jerk is unobtainable. The most common c

ause of compression or injury to the sciatic nerve in this region is trauma, whi ch includes hip dislocation, fracture, or replacement. Other etiologies include wayward buttock injections, compression by external sources (e.g., prolonged be d rest), and any deep-seated mass in the pelvis (e.g., hematoma). (Choice C) A patient with an obturator nerve injury will present with pain, weak ness in leg adduction, and sensory loss over a small area in the medial thigh. Obturator neuropathy is often secondary to pelvic trauma or surgery. (Choice D) A patient with a common peroneal nerve injury will usually present wi th an acute foot drop accompanied by weakness in foot dorsiflexion and eversion. In addition, the patient may also complain of paresthesias and/or sensory loss over the dorsum of the foot and lateral shin (superficial peroneal nerve territ ory). The injury is usually located at the knee, on the lateral aspect of the f ibular head. Educational Objective: Femoral nerve injury is characterized by the following physical findings: inabil ity to extend the knee, loss of knee jerk reflex, and sensory loss over the ante rior and medial aspects of the thigh, medial aspect of shin, and arch of the foo t. 29. An asymptomatic, 21-year-old Caucasian woman comes to the physician for a ro utine health maintenance examination. She has no other medical problems. She d oes not use tobacco. She does not drink alcohol. Her family history is not sig nificant. She has no medications. Her vital signs are within normal limits. E xamination shows no abnormalities. Her annual pap smear reveals low-grade squam ous intraepithelial lesions (LSIL). Satisfactory colposcopy examination confirm s CIN 1. Which of the following is the most appropriate next step in the manage ment of this patient? A. B. C. D. E. Repeat Pap in 6 months Cryosurgery Laser ablation Cold knife conization LEEP A

Answer:

Most of the low-grade squamous intraepithelial lesions (LSIL) or low-grade cervi cal intraepithelial neoplasia (CIN I) regress spontaneously; therefore, expectan t management is preferred for biopsy proven CIN 1 with satisfactory colposcopic examination. A colposcopic examination is satisfactory when an entire lesion an d a transformation zone are visualized. Expectant management includes repeat cy tology at 6 and 12 months, or HPV DNA testing at 12 months. If there is progres sion during the follow-up, or lesions are persistent after one year, treatment i s indicated. In the above patient, colposcopy examination is satisfactory; ther efore, the next best step should be expectant management. (Choices B, C, D, and E) All other choices would have been appropriate if colpos copic examination was unsatisfactory. When the colposcopic exam is unsatisfacto ry, the next step is always excision as this allows for histological examination . Even if colposcopic examination is satisfactory, treatment may be indicated i f the patient is anxious about her disease, or if she seems to be non-compliant with follow-up. If the patient opts for treatment, available modalities are eit her ablation or excision. Ablation can be done with cryosurgery or laser, and e xcision can be done with knife conization, laser conization, or Loop Electrosurg ical Excision Procedure (LEEP).

Educational Objective: Expectant management is the preferred option for biopsy proven CIN 1 with satisf actory colposcopic examination. 43% of people answered this question correctly. 30. A 43-year-old African female is recently diagnosed with adult polycystic kid ney disease. While explaining the long-term management of the disease, her phys ician addresses the issue of different modalities of dialysis therapy. He asks the patient which modality (hemodialysis or peritoneal dialysis) she would choos e if the need arises in the future. The patient shows a preference for peritone al dialysis. Which of the following investigations is needed before the patient can be considered as a candidate for peritoneal dialysis? A. B. C. D. E. CT scan of the head Echocardiography Chest x-ray Intravenous pyelography Total colonoscopy E

Answer:

Patients with adult polycystic kidney disease are predisposed to the following e xtra-renal manifestations: Hepatic, pancreatic, splenic, and pulmonary cysts Cerebral aneurysms Aortic aneurysm Colonic diverticula Mitral valve prolapse Inguinal and abdominal hernias Before considering peritoneal dialysis as a treatment option in patients with ad ult polycystic kidney disease, it is important to rule out diverticulosis, which can potentially complicate peritoneal dialysis. This can be done by performing a total colonoscopy. (Choices A, B, C, and D) Other investigations may help detect other extra-renal manifestations of adult polycystic kidney disease, but none of these will affect the decision to forego peritoneal dialysis. Educational Objective: Before considering peritoneal dialysis as a treatment option in patients with ad ult polycystic kidney disease, it is important to rule out diverticulosis. 34% of people answered this question correctly. 31. A large-scale clinical trial was conducted to assess the effect of a multi-v itamin supplement on the risk of future cardiovascular events. The outcomes of the study were cardiovascular mortality, non-fatal myocardial infarction, and co ronary revascularization procedures. According to the study results, the overal l relative risk of having the cardiovascular outcomes for the placebo group comp ared to the treatment group was 1.5, p = 0.30, although the relative risk for on ly the fifth year of follow-up was 2.05, p = 0.01. Survival curves for the two groups were parallel during the first three years of observation, but began to s eparate in the third year, favoring the treatment group. Which of the following statements is true concerning the study results given above? A. Multi-vitamin use seems to be ineffective in preventing cardiovascular event

s B. C. D. E. Inappropriate selection of the study Latent period can be demonstrated on The follow-up period is too long for The sample size is not large enough, C subjects may be present the survival plot such a study and the measure of outcome is unstable

Answer:

The concept of latent period is demonstrated in this case. Latency assumption i s a very important issue to consider in chronic disease epidemiology. The laten t period from getting exposed to developing an outcome is relatively short in in fectious diseases. On the other hand, chronic diseases (e.g., cancer or coronar y artery disease) may have a very long latent period. In this case, at least th ree years of continuous exposure to multivitamins were required to reveal a prot ective effect of the exposure on the cardiovascular outcomes. On the survival p lot, you can clearly see that the survival curves run parallel to each other (la tent period) and then begin to separate at the third year of follow-up, favoring the treatment group. (Choice D) An extended period of continuous exposure may be necessary to affect the outcome of this study. (Choice A) Overall, the relative risk (RR) is not statistically significant, bec ause it is ?diluted? by the earlier years of latency; however, the RR for only t he fifth year of follow-up clearly demonstrates the beneficiary effect of the th erapy. (Choice B) The latent period demonstrated in this study is a natural phenomenon, and does not imply selection bias. (Choice E) The sample size is large enough to demonstrate the efficacy of therap y. Educational Objective: The concept of latent period is an important issue in chronic disease epidemiolo gy. The exposure must be continuous for a certain period of time (called latent period) in order to influence the outcome. 50% of people answered this question correctly. 32. A 66-year-old male with chronic rheumatoid arthritis comes to the emergency department with fever and cough productive of yellow sputum for the past ten hou rs. He has had rheumatoid arthritis for the past twelve years. He has been tak ing methotrexate (5 mg/wk), prednisone (7.5 mg/d), and NSAIDs (as needed) for th e last several years. Physical examination reveals a sick-appearing male with c ushingoid features. His blood pressure is 90/70 mm Hg, heart rate is 110/min, t emperature is 102 F(38.9C) and respirations are 24/min. His oxygen saturation b y pulse oximetry is 92% on 2 liters of intranasal oxygen. His mucous membranes are moist. There is no pallor or icterus. Lung examination reveals crackles on the left base. The other systems are normal. Laboratory investigations reveal an increase in the total WBC count with a left shift. His basic serum chemistr y profile is normal, except for a serum sodium level of 131 mEq/L and BUN level of 27 mg/dL. Chest x-ray reveals a left lower lobe infiltrate. EKG reveals sin us tachycardia with nonspecific ST-T wave changes. What is the most likely cause of hypotension in this patient? A. Sepsis B. Aldosterone deficiency

C. Cortisol deficiency D. ACTH deficiency E. Cardiogenic shock Answer: C

This patient's history of long-term exposure to a glucocorticoid drug makes him likely to have chronic suppression of the hypothalamic-pituitary-adrenal (HPA) a xis, which may result in secondary or tertiary adrenal insufficiency due to the decrease in the plasma ACTH and cortisol levels, respectively. Since ACTH does not control the secretion of mineralocorticoids from the zona glomerulosa of the adrenal glands, patients have normal plasma aldosterone levels and are normoten sive; however, these patients can develop vascular collapse under stressful situ ations such as an infection due to a decrease in cortisol, which is the hormone responsible for the maintenance of vascular tone. Furthermore, since cortisol p romotes the synthesis of catecholamines in the adrenal medulla, cortisol-deficie nt patients are unable to mount a good pressor response during stress. The hypo tension in this patient should respond to fluid repletion and a stress dose of c orticosteroids. (Choice A) Although this patient may have sepsis, he is more likely to have hypo tension secondary to a suppressed HPA axis. When his blood pressure does not re spond to adequate hydration and a stress dose of glucocorticoids, sepsis should be considered. (Choice B) The secretion of aldosterone is not altered in patients with secondar y or tertiary adrenal failure because aldosterone is not primarily regulated by ACTH. (Choice D) ACTH doesn?t have any direct effects on blood pressure or the vessel tone. ACTH mainly works through the release of cortisol from the adrenal gland. (Choice E) This clinical picture and insignificant EKG changes make the diagnosi s of cardiogenic shock unlikely. Educational Objective: Patients with chronic suppression of hypothalamic-pituitary-adrenal axis can hav e hypotension during acute infections. Hypotension in these patients usually re sponds to administration of a stress dose of glucocorticoids and hydration. In secondary and tertiary adrenal failure, there are no significant abnormal minera locorticoid levels. 27% of people answered this question correctly.

33. A 66-year-old male with chronic rheumatoid arthritis comes to the emergency department with fever and cough productive of yellow sputum for the past ten hou rs. He has had rheumatoid arthritis for the past twelve years. He has been tak ing methotrexate (5 mg/wk), prednisone (7.5 mg/d), and NSAIDs (as needed) for th e last several years. Physical examination reveals a sick-appearing male with c ushingoid features. His blood pressure is 90/70 mm Hg, heart rate is 110/min, t emperature is 102 F(38.9C) and respirations are 24/min. His oxygen saturation b y pulse oximetry is 92% on 2 liters of intranasal oxygen. His mucous membranes are moist. There is no pallor or icterus. Lung examination reveals crackles on the left base. The other systems are normal. Laboratory investigations reveal an increase in the total WBC count with a left shift. His basic serum chemistr y profile is normal, except for a serum sodium level of 131 mEq/L and BUN level of 27 mg/dL. Chest x-ray reveals a left lower lobe infiltrate. EKG reveals sin us tachycardia with nonspecific ST-T wave changes.

Intravenous fluids and antibiotics were administered after drawing blood culture s. What is the most appropriate next step in management? A. Place arterial line and start dopamine. B. Administer intravenous methylprednisone. C. Administer dexamethasone and perform cosyntropin stimulation test. D. Start fludrocortisone. E. Perform dexamethasone suppression test. Answer: C

The primary goals of therapy in patients with acute adrenal insufficiency are to reverse the hypotension, correct the electrolyte abnormalities, and replace cor tisol. Prompt administration of intravenous steroids is indicated without waiti ng for the confirmation of the diagnosis. Intravenous dexamathasone is preferre d because it is long-acting and does not interfere with the measurement of serum or urinary steroids during subsequent cosyntropin (ACTH) stimulation test. (Choice D) Mineralocorticoid treatment is not necessary since it takes a long ti me to show its sodium-retaining effects. The same effect can be achieved with n ormal saline very quickly. (Choice E) Dexamethasone suppression test is used for Cushing?s syndrome. Educational Objective: Prompt administration of dexamethasone followed by ACTH stimulation test are ind icated in patients with suspected acute adrenal insufficiency. 20% of people answered this question correctly. 34. A 38-year-old Caucasian primigravida presents to the physician's office at 2 0 weeks gestation for prenatal counseling. She is concerned about the risk of D own syndrome, and asks about the measures to diagnose it early. The physician e xplains that triple screening may detect up to 50% of the cases of chromosomal a bnormalities, and that amniocentesis may detect approximately 90% of the cases. She decides not to undergo any test. Eighteen weeks later, she gives birth to a child with Down syndrome. When the physician compared amniocentesis to triple screening, which of the following did he emphasize? A. B. C. D. E. Increase Increase Increase Increase Increase E in in in in in false negatives false positives positive predictive value negative predictive value sensitivity

Answer:

Before expounding on the characteristics of a diagnostic test, it is important t o correctly define the following terms: true positive, false positive, true nega tive, and false negative. True positive occurs when a patient who really has th e disease obtains a positive test result (using the diagnostic test). False po sitive occurs when a patient who really does not have the disease obtains a posi tive test result. True negative occurs when a patient who really does not have the disease obtains a negative test result. False negative occurs when a patien t who really has the disease obtains a negative test result. The most commonly discussed characteristic of a diagnostic test is its sensitivi ty. The sensitivity of a test determines the capacity of the test to correctly diagnose a patient with the disease. It is obtained by dividing the number of

true positives by the number of people who have the disease (true positives + fa lse negatives). In this case, the physician explained to the patient in layman? s terms that triple screening has a sensitivity of 50% in detecting chromosome a bnormalities, while amniocentesis has a sensitivity of 90%. (Choice C) The predictive value of a test gives the probability of a disease aft er running a test and getting the results; for example, if triple screening turn ed out to be positive you should discuss the probability of having a given chrom osome abnormality given positive test result, i.e., positive predictive value . (Choice D) The negative predictive value of the test determines the capacity of the test to correctly give a negative result. It is obtained by dividing the nu mber of true negatives by the number of people who obtained a negative result (t rue negatives + false negatives). (Choices A and B) Based on the above values, amniocentesis gives less false nega tives and more true positives. Educational Objective: Sensitivity of a test is the probability of detecting the disease if it is prese nt. 35. A 41-year-old female is seen for amenorrhea of eight months duration. Her p regnancy tests have been negative during this period. She denies headaches, vis ual change, galactorrhea, hot flashes, dyspareunia, weight change, or loss of se condary sexual characters. Her past medical history and family history are unre markable. She is not on any medications, including over-the-counter medications . She denies the use of tobacco, alcohol or intravenous drugs. The physical ex amination is unrevealing. Her routine labs are within normal limits. Her hormo ne profile reveals a prolactin level of 50 ng/mL (normal 5-20 ng/ml). Her LH is undetectable, and FSH is low normal. The level of serum a-subunits is markedly increased. Her serum IGF1 levels are normal. MRI of the pituitary shows a 2 c m pituitary tumor with suprasellar extension. What is the most appropriate mana gement of this patient? A. B. C. D. E. Transphenoidal pituitary surgery Estrogen-progesterone cyclically Bromocriptine orally Pituitary radiation Octreotide A

Answer:

The patient has hypogonadism with suppressed LH and FSH levels, and increased asubunits. This is classically seen in a nonfunctioning pituitary adenoma, which usually arises from the gonadotropin-secreting cells (gonadotrophs) in the pitu itary gland. Gonadotrophs secrete both LH and FSH, which are dimeric glycoprotein hormones. They are composed of a common a-subunit and hormone-specific b-subunit. TSH and Beta-HCG are also dimeric glycoprotein hormones similar to LH and FSH, which ha ve common a and hormone-specific beta sub-units. Although these tumors arise fr om the gonadotrophs, they do not usually overproduce intact LH and FSH, but gene rally overproduce the a-subunit. Occasionally, intact FSH is overproduced by th ese tumors. LH overproduction is exceedingly rare. The increase in prolactin l evel in this patient is small, and is possibly due to compression of the pituita ry stalk by the tumor. The first-line therapy for most nonsecretory pituitary adenomas is transphenoida

l surgery. Patients might regain their normal gonadal functions after resection . (Choice B) Treatment with estrogen and progesterone will improve her amenorrhea, but will not have any effect on the large tumor. (Choice C) Bromocriptine, a dopaminergic receptor agonist, has minimal effects o n pituitary tumors other than prolactin and growth hormone-secreting tumors. (Choice D) Pituitary radiation is almost never the first choice in the treatment of pituitary tumors due to its delayed effect and risk of hypopituitarism. (Choice E) Octreotide, a somatostatin analogue, acts on somatostatin receptors o n the nonfunctioning pituitary adenomas. It is not the preferred primary therap y because the decrease in tumor size with octreotide is usually minimal. Educational Objective: Nonfunctioning pituitary adenomas generally arise from gonadotropin-secreting ce lls of the pituitary glands. Patients usually present with hypogonadism and low gonadotropin levels. The serum a-subunit levels are characteristically increas ed. Large tumors can have local, compressive symptoms. The primary modality for the treatment is transphenoidal surgery. 34% of people answered this question correctly. 36. A 41-year-old female is seen for amenorrhea of eight months duration. Her p regnancy tests have been negative during this period. She denies headaches, vis ual change, galactorrhea, hot flashes, dyspareunia, weight change, or loss of se condary sexual characters. Her past medical history and family history are unre markable. She is not on any medications, including over-the-counter medications . She denies the use of tobacco, alcohol or intravenous drugs. The physical exa mination is unrevealing. Her routine labs are within normal limits. Her hormone profile reveals a prolactin level of 50 ng/mL (normal 5-20 ng/ml). Her LH is un detectable, and FSH is low normal. The level of serum a-subunits is markedly in creased. Her serum IGF1 levels are normal. MRI of the pituitary shows a 2 cm p ituitary tumor with suprasellar extension. What is the most appropriate managem ent of this patient? A. B. C. D. E. Transphenoidal pituitary surgery Estrogen-progesterone cyclically Bromocriptine orally Pituitary radiation Octreotide A

Answer:

The patient has hypogonadism with suppressed LH and FSH levels, and increased asubunits. This is classically seen in a nonfunctioning pituitary adenoma, which usually arises from the gonadotropin-secreting cells (gonadotrophs) in the pitu itary gland. Gonadotrophs secrete both LH and FSH, which are dimeric glycoprotein hormones. They are composed of a common a-subunit and hormone-specific b-subunit. TSH and Beta-HCG are also dimeric glycoprotein hormones similar to LH and FSH, which ha ve common a and hormone-specific beta sub-units. Although these tumors arise fr om the gonadotrophs, they do not usually overproduce intact LH and FSH, but gene rally overproduce the a-subunit. Occasionally, intact FSH is overproduced by th ese tumors. LH overproduction is exceedingly rare. The increase in prolactin l evel in this patient is small, and is possibly due to compression of the pituita

ry stalk by the tumor. The first-line therapy for most nonsecretory pituitary adenomas is transphenoida l surgery. Patients might regain their normal gonadal functions after resection . (Choice B) Treatment with estrogen and progesterone will improve her amenorrhea, but will not have any effect on the large tumor. (Choice C) Bromocriptine, a dopaminergic receptor agonist, has minimal effects o n pituitary tumors other than prolactin and growth hormone-secreting tumors. (Choice D) Pituitary radiation is almost never the first choice in the treatment of pituitary tumors due to its delayed effect and risk of hypopituitarism. (Choice E) Octreotide, a somatostatin analogue, acts on somatostatin receptors o n the nonfunctioning pituitary adenomas. It is not the preferred primary therap y because the decrease in tumor size with octreotide is usually minimal. Educational Objective: Nonfunctioning pituitary adenomas generally arise from gonadotropin-secreting ce lls of the pituitary glands. Patients usually present with hypogonadism and low gonadotropin levels. The serum a-subunit levels are characteristically increase d. Large tumors can have local, compressive symptoms. The primary modality for t he treatment is transphenoidal surgery. 34% of people answered this question correctly. 37. A 41-year-old female is seen for amenorrhea of eight months duration. Her p regnancy tests have been negative during this period. She denies headaches, vis ual change, galactorrhea, hot flashes, dyspareunia, weight change, or loss of se condary sexual characters. Her past medical history and family history are unre markable. She is not on any medications, including over-the-counter medications . She denies the use of tobacco, alcohol or intravenous drugs. The physical ex amination is unrevealing. Her routine labs are within normal limits. Her hormo ne profile reveals a prolactin level of 50 ng/mL (normal 5-20 ng/ml). Her LH is undetectable, and FSH is low normal. The level of serum a-subunits is markedly increased. Her serum IGF1 levels are normal. MRI of the pituitary shows a 2 c m pituitary tumor with suprasellar extension. What is the most appropriate mana gement of this patient? A. B. C. D. E. Transphenoidal pituitary surgery Estrogen-progesterone cyclically Bromocriptine orally Pituitary radiation Octreotide A

Answer:

The patient has hypogonadism with suppressed LH and FSH levels, and increased asubunits. This is classically seen in a nonfunctioning pituitary adenoma, which usually arises from the gonadotropin-secreting cells (gonadotrophs) in the pitu itary gland. Gonadotrophs secrete both LH and FSH, which are dimeric glycoprotein hormones. They are composed of a common a-subunit and hormone-specific b-subunit. TSH and Beta-HCG are also dimeric glycoprotein hormones similar to LH and FSH, which ha ve common a and hormone-specific beta sub-units. Although these tumors arise fr om the gonadotrophs, they do not usually overproduce intact LH and FSH, but gene

rally overproduce the a-subunit. Occasionally, intact FSH is overproduced by th ese tumors. LH overproduction is exceedingly rare. The increase in prolactin l evel in this patient is small, and is possibly due to compression of the pituita ry stalk by the tumor. The first-line therapy for most nonsecretory pituitary adenomas is transphenoida l surgery. Patients might regain their normal gonadal functions after resection . (Choice B) Treatment with estrogen and progesterone will improve her amenorrhea, but will not have any effect on the large tumor. (Choice C) Bromocriptine, a dopaminergic receptor agonist, has minimal effects o n pituitary tumors other than prolactin and growth hormone-secreting tumors. (Choice D) Pituitary radiation is almost never the first choice in the treatment of pituitary tumors due to its delayed effect and risk of hypopituitarism. (Choice E) Octreotide, a somatostatin analogue, acts on somatostatin receptors o n the nonfunctioning pituitary adenomas. It is not the preferred primary therap y because the decrease in tumor size with octreotide is usually minimal. Educational Objective: Nonfunctioning pituitary adenomas generally arise from gonadotropin-secreting ce lls of the pituitary glands. Patients usually present with hypogonadism and low gonadotropin levels. The serum a-subunit levels are characteristically increas ed. Large tumors can have local, compressive symptoms. The primary modality fo r the treatment is transphenoidal surgery. 34% of people answered this question correctly. 38. A 33-year-old Caucasian male presents to the emergency department with two e pisodes of coffee-ground colored vomiting. He denies ever having such symptoms in the past. He also denies any abdominal pain, dizziness, syncope and black st ool. His past medical history is insignificant. He does not smoke or consume a lcohol. His blood pressure is 120/72 mmHg while supine and 122/70 mmHg while st anding. His heart rate is 90/min. The physical examination is within normal li mits. Endoscopy reveals a single small longitudinal tear at the gastro-esophage al junction that is not actively bleeding. Which of the following is the best management for this patient? A. Observation and supportive care B. Thermal coagulation C. Sclerotherapy D. Band ligation E. IV infusion of vasopressin Answer: A This patient presents with upper GI hemorrhage caused by a Mallory-Weiss tear. The "classic" presentation of hematemesis preceded by a bout of retching/vomitin g only occurs in 30% of patients. Endoscopy is the gold standard in establishing the diagnosis. This procedure typically reveals a single longitudinal tear at the gastro-esophageal junction. In patients with Mallory-Weiss tear who are not actively bleeding (such as the patient in this case), observation and supportiv e care are typically necessary. (Choices B, C, and D) Endoscopy also allows for therapeutic interventions (i.e., thermal coagulation, sclerotherapy and band ligation) to stop active bleeding.

(Choice E) IV infusion of vasopressin, esophageal balloon tamponade, and angiogr aphic arterial embolization have been used occasionally to control severe or ref ractory hemorrhage. Educational Objective: In patients with Mallory-Weiss tear who are not actively bleeding, observation and supportive care are typically necessary. 39. A 33-year-old Caucasian male presents to the emergency department with two e pisodes of coffee-ground colored vomiting. He denies ever having such symptoms in the past. He also denies any abdominal pain, dizziness, syncope and black st ool. His past medical history is insignificant. He does not smoke or consume a lcohol. His blood pressure is 120/72 mmHg while supine and 122/70 mmHg while st anding. His heart rate is 90/min. The physical examination is within normal li mits. Endoscopy reveals a single small longitudinal tear at the gastro-esophage al junction that is not actively bleeding Which of the following is the most likely associated finding in this type of pat ient? A. Increased lower esophageal sphincter (LES) tone B. Hiatal hernia C. Gastric atrophy D. Peptic ulcer disease E. Esophageal varicosities Answer: B Hiatal hernia is the most well known anatomical predisposing factor for MalloryWeiss syndrome. According to different sources, it is present in 40-100% of pat ients with this syndrome. During retching or vomiting, the transmural pressure gradient is greater within the hernia than the rest of the stomach, thereby maki ng this location the most likely to sustain a tear. Other precipitating factors include retching, vomiting, straining, hiccuping, coughing, primal scream thera py, blunt abdominal trauma, cardiopulmonary resuscitation, and diagnostic or the rapeutic manipulation (e.g., endoscopy). (Choice E) Esophageal varicosities may be revealed in chronic alcoholics who pre sent with this syndrome, but these are not predisposing factors to tears per se. (Choices C, D, and A) No significant association is present with gastric atrophy , peptic ulcer disease and achalasia. Educational Objective: Hiatal hernia is present in 40-100% of patients with Mallory-Weiss syndrome.

40. A 54-year-old Hispanic male comes for a routine follow-up visit and states t hat he might be losing weight, particularly from his extremities. He has HIV in fection, for which he has been receiving highly active antiretroviral therapy (H AART) for one year. Following his HAART treatment, he showed a significant impr ovement in his CD4 lymphocyte count and a decrease in his viral titers. His pas t medical history, aside from his HIV infection, is unremarkable. He does not h ave hepatitis C or hepatitis B. His father had coronary artery disease and died of a heart attack at the age of 55 years. He does not drink alcohol. He has s moked one pack of cigarettes daily for the last 28 years. He has a history of i ntravenous drug abuse in the past, but he is currently not using any recreationa l drugs. On examination, he weighs 168 pounds (his weight one year ago was 160 pounds). His height is 5?9??(175cm). He has fat tissue depositions on the back

of his neck and on his abdomen. His extremities and face appear to be thinned out. The rest of the physical examination is unremarkable, except for minimally palpable hepatomegaly. Lab investigations reveal a normal complete blood count and routine basic chemistry. His lipid profile revealed a total cholesterol le vel of 280 mg/dL, triglyceride level of 530 mg/dL, and high-density lipoprotein level of 29 mg/dL. His fasting blood sugar is 140 mg/dL and LFTs shows mild ele vations of AST and ALT levels. What is the next best step in the management of this patient? A. B. C. D. Start nicotinic acid therapy. Start gemfibrozil therapy. Start atorvastatin therapy. Suggest NCEP ATP III dietary restrictions with other lifestyle modifications.

E. Measure 24-hour urinary cortisol levels. Answer: B

This is a classic presentation of HIV lipodystrophy occurring after HAART. Alth ough the mechanism of HIV lipodystrophy is unclear, protease inhibitors are most likely responsible. Elevated liver enzymes are most likely due to steatosis. In all patients with hyperlipidemia, including those with HIV lipodystrophy, die tary restrictions and lifestyle modifications are necessary. In this patient, d rug therapy is also indicated because of severe hyperlipidemia (triglycerides 53 0 mg/dL) and the presence of risk factors for a coronary event (male sex, positi ve family history, smoker, impaired glucose tolerance). His main lipid abnormal ity appears to be hypertriglyceridemia. His increased total cholesterol is most likely due to an increase in VLDL (rather than LDL) cholesterol. Gemfibrozil i s a fibric acid derivative which is mainly used for hypertriglyceridemia but can also cause a modest reduction of LDL cholesterol levels. It is preferred over other fibric acid derivatives because it has been extensively studied in patient s with HIV infection. The drug interactions with gemfibrozil are less than thos e with other fibric acid derivatives. (Choice A) Nicotinic acid is also likely to improve his lipid profile; however, it will not help to treat the glucose intolerance, and is therefore not preferre d over gemfibrozil for this patient. (Choice C) Atorvastatin is not the first line therapy for patients with hypertri glyceridemia. It primarily reduces serum LDL cholesterol levels by inhibiting H MG-CoA reductase. Although it decreases triglycerides levels and increases HDL levels, the decrease in triglyceride levels is mainly seen in patients who have elevated LDL levels. Since the increase in total cholesterol in this patient is most likely due to the elevation of VLDL cholesterol, treatment with atorvastat in will not be beneficial. (Choice D) Dietary modifications alone usually lead to an improvement in the lip id profile; however, this patient has severe triglyceride elevation. Drug thera py is necessary in addition to dietary and lifestyle modifications. (Choice E) Although central deposition of fat, a buffalo hump , and impaired glu cose tolerance could suggest a diagnosis of Cushing?s syndrome in this patient, this presentation is classic and therefore more suggestive of HIV lipodystrophy. One of the important differences between Cushing?s syndrome and HIV lipodystro phy is the absence or loss of fat from the face. Cushing?s syndrome presents wit h rounding of the face, accompanied by facial flushing called "moon facies". In patients with HIV lipodystrophy, the face is thinned out. Measurement of 24-ho ur urinary cortisol will be normal in patients with HIV lipodystrophy and will b e elevated in patients with Cushing?s syndrome.

Educational Objective: HAART can lead to lipodystrophic syndrome (redistribution of fat and insulin res istance) and significant hyperlipidemia. Gemfibrozil is the treatment of choice for hypertriglyceridemia seen in patients with HIV lipodystrophy. The following Vignette applies to the next 2 items

41. A 43-year-old HIV-positive Caucasian male with a history of intravenous drug abuse was admitted to the hospital for nausea and vomiting and severe pain in t he mouth, throat, and epigastric region. He had been diagnosed with AIDS four y ears ago after presenting with pneumocystis carinii pneumonia, which resolved wi th treatment. Since his diagnosis he has been on prophylactic antibiotics and h ighly active antiretroviral therapy, which has been effective in slowing the det erioration of his immune function. After admission, it proved necessary to begi n total parenteral nutrition via a central venous catheter because of his inabil ity to take in food or liquid by mouth. When he then became increasingly febril e, hypotensive, tachycardic, and tachypneic, he was transferred to the intensive care unit. While there he began to complain of significant pain and sensitivit y to light in his right eye. On funduscopic examination, several large, glisten ing, off-white lesions with indistinct borders are seen. The lesions are threedimensional and appear to extend from the chorioretinal surface into the vitreou s. A vitreous haze is present. Visual acuity is decreased. What is the most likely cause of his ocular pain? A. Mycobacterium avium complex B. Infective endocarditis C. Cryptococcosis D. Candidiasis E. Aspergillosis Answer: D

Oropharyngeal and esophageal candidiasis arises commonly in HIV-positive patient s and can disseminate widely to multiple organs, including the eye. Endogenous candida endophthalmitis (Choice D) is caused by hematogenous seeding of the eye with Candida species (especially C. albicans), and is found in 10-28% of patient s with candidiasis. The presence of candida endophthalmitis is a marker for wid espread disseminated candidiasis and should not be taken lightly. The condition is frequently characterized by ocular pain, photophobia, scotomas, and fever. Risk factors that place patients at greatest risk for developing candida endopht halmitis include central venous catheters, total parenteral nutrition, broad-spe ctrum antibiotic therapy, prior abdominal surgery, neutropenia, corticosteroid t herapy, and injection drug abuse. Mycobacterium avium complex (Choice A) is characterized by fever, night sweats, fatigue, shortness of breath, abdominal pain, diarrhea, weight loss, and lymphad enopathy. Ocular manifestations are not expected. Infective endocarditis (Choice B) is often subtle and nonspecific in presentatio n. It may be characterized by fever, fatigue, anorexia, back pain, and weight l oss. One rarely seen ocular manifestation of infective endocarditis is the Roth spot, which is a retinal hemorrhage with a pale center. Pulmonary cryptococcosis (Choice C) is characterized by fever, malaise, cough, d yspnea, and pleuritic pain. CNS cryptococcosis is characterized by headache, al tered mental status, nausea, and vomiting. Ocular manifestations of CNS cryptoc occosis are uncommon, but can include papilledema, optic neuritis, and chorioret initis.

Invasive aspergillosis (Choice E) can occur in immunosuppressed patients and is characterized by fever, cough, dyspnea, pleuritic chest pain, and occasionally h emoptysis. Aspergillus endophthalmitis results from hematogenous spread and can cause eye pain and visual changes. Educational Objective: Risk factors that place patients at greatest risk for developing candida endopht halmitis include central venous catheters, total parenteral nutrition, broad-spe ctrum antibiotic therapy, prior abdominal surgery, neutropenia, corticosteroid t herapy, and injection drug abuse. 41% of people answered this question correctly.

42. A 43-year-old HIV-positive Caucasian male with a history of intravenous drug abuse was admitted to the hospital for nausea and vomiting and severe pain in t he mouth, throat, and epigastric region. He had been diagnosed with AIDS four y ears ago after presenting with pneumocystis carinii pneumonia, which resolved wi th treatment. Since his diagnosis he has been on prophylactic antibiotics and h ighly active antiretroviral therapy, which has been effective in slowing the det erioration of his immune function. After admission, it proved necessary to begi n total parenteral nutrition via a central venous catheter because of his inabil ity to take in food or liquid by mouth. When he then became increasingly febril e, hypotensive, tachycardic, and tachypneic, he was transferred to the intensive care unit. While there he began to complain of significant pain and sensitivit y to light in his right eye. On funduscopic examination, several large, glisten ing, off-white lesions with indistinct borders are seen. The lesions are threedimensional and appear to extend from the chorioretinal surface into the vitreou s. A vitreous haze is present. Visual acuity is decreased. Given the patient?s condition, which of the following treatment options is likel y to be of greatest benefit? A. Vitrectomy and systemic ketoconazole B. Vitrectomy and systemic amphotericin B C. Intravenous vancomycin and gentamycin D. Systemic amphotericin B only E. Clarithromycin and rifabutin Answer: B

Patients with candida endophthalmitis who have chorioretinitis with vitreal invo lvement should be treated with vitrectomy and systemic antifungal therapy with a mphotericin B (Choice B) and/or fluconazole. An early vitrectomy improves the l ikelihood of a positive outcome, and intravitreal injection of amphotericin B ma y be of help. Prompt diagnosis and treatment of candida endophthalmitis is esse ntial, as the condition can worsen quickly. Although ketoconazole (Choices A) can treat candidiasis, it does not attain suff icient concentration within the eye and is therefore not an ideal choice in this case. Systemic amphotericin B (Choice D) attains sufficient concentration within the c horoid and retina, but not within the vitreous body. Patients who have candida endophthalmitis with vitreal involvement will therefore require vitrectomy in co

njunction with systemic antifungal treatment. (Choice C) Combination of vancomycin and gentamycin is used in patients with inf ective endocarditis (in IV drug users) as empirical therapy. Educational Objective: Candida endophthalmitis with vitreal involvement should be treated with vitrecto my and systemic antifungal therapy (amphotericin B and/or fluconazole). 45% of people answered this question correctly. 43. A 56-year-old male accountant is brought to the emergency department by para medics because of a sudden onset of severe crushing chest pain and dizziness. T he pain started while he was in a meeting with some of his clients. He has a hi story of longstanding hypertension, gastroesophageal reflux disease, hyperlipide mia, and chronic stable angina. He occasionally gets substernal chest pressure with brisk walking and running, and this is completely resolved by sublingual ni trates. He is a lifelong smoker and currently smokes two packs of cigarettes a day. His other medications include aspirin, hydrochlorothiazide, metoprolol and atorvastatin. During this episode, he took four sublingual nitroglycerine tabl ets in the first 10 minutes, which did not change the character and severity of the pain. His temperature is 36.7C(98F), blood pressure is 160/86 mm Hg, pulse is 80/min, and respirations are 16/min. Oxygen saturation is 94% on room air. An initial EKG and chest x-ray done in the emergency department are normal. Whi ch of the following is the most appropriate statement to tell the patient? A. B. C. D. E. You You You You You may may may may may C be having an acute myocardial infarction. have a perforated peptic ulcer. have an aortic dissection. have a pulmonary embolism. have a tension pneumothorax.

Answer:

Aortic dissection is an uncommon but potentially life threatening cause of sever e and sudden chest pain. It has the same predisposing risk factors as an acute coronary syndrome (unstable angina and myocardial infarction), including age gre ater than 55 years, hypertension, hyperlipidemia, and chronic smoking. The diag nosis of aortic dissection is usually based on the history and the physical exam ination findings. Patients with acute aortic dissection typically present with a sudden onset of severe or "tearing" chest pain radiating to the back. The pai n is usually not relieved by sublingual nitroglycerine. Other clinical clues fa voring the diagnosis of an aortic dissection include the blood pressure differen ce in the two arms, an early diastolic murmur (reflecting aortic insufficiency), and the presence of mediastinal widening on chest x-ray. (Choice A) The patient certainly has risk factors and a history of coronary arte ry disease based on his symptoms of exertional angina. A sudden onset of severe chest pain, unrelieved by nitrates, can be due to an acute myocardial infarctio n; however, it would be unusual to have a normal EKG in a patient with acute MI while he is having active chest pain. (Choice B) Peptic ulcer perforation is an unlikely diagnosis in the absence of a ny abnormal abdominal exam findings. Furthermore, a chest-x ray/erect abdominal -x ray usually shows air under the diaphragm. (Choice D) Pulmonary embolism usually presents with pleuritic chest pain that is less severe in intensity, and is associated with shortness of breath and hypoxi a.

(Choice E) Tension pneumothorax presents as a sudden onset of shortness of breat h, hypoxia, absent airflow, and hyperresonance to percussion on the affected sid e. The patient s chest x-ray and examination findings are not consistent with tens ion pneumothorax. Educational Objective: Aortic dissection is a catastrophic condition, and should be suspected from the initial history and physical examination findings. 44. A 35-year-old woman comes to her physician after she palpated a lump in her right breast. She doesn?t have any other complaints. She hasn?t seen a doctor for ten years. She states that she regularly performs a self-breast exam after menses. Her past medical history is unremarkable. Her mother died of breast ca ncer at the age of 40. Inspection of her breasts is normal. A 1 x 1cm rubbery, firm, freely mobile round mass is palpated in the upper, outer quadrant. No ax illary lymph nodes are palpated. What is the best next action? A. B. C. D. E. Observation Ultrasonography Excisional biopsy Fine needle aspiration Mammography E

Answer:

A baseline mammogram at age 35 is recommended for any woman who has an increased risk for breast cancer. All patients with breast lumps after the age of 35 sho uld be evaluated using mammography, especially if they have a family history of breast cancer. The findings on the physical exam are consistent with a fibroadenoma. Apart fro m the fibroadenoma, this patient might have other anomalies imperceptible on phy sical exam that might be seen on the mammogram, which is more superior for this purpose than an ultrasound (Choice B). It is best to make further decisions for this patient?s management after the mammogram is obtained. (Choices A, C, and D) Although the remaining choices are also appropriate for th e management of breast masses, mammography is the best initial step for this pat ient. Educational Objective: All patients with breast lumps after the age of 35 should be evaluated with mamm ography, especially if they have a family history of breast cancer. 32% of people answered this question correctly.

45. A 46-year-old man is admitted to the hospital with complaints of constant mi d-abdominal pain for the past five days. He has a history of intravenous drug a buse, alcoholism, and chronic pancreatitis. He continues to drink heavily. He has had three recent admissions for pancreatitis in the past four months. You c onsult a gastroenterologist, a close friend of yours, for a possible endoscopic retrograde cholangiopancreatography. He has also known the patient from his pre vious admissions. After the procedure, he calls and tells you, "I do not want t o get involved in this patient?s care. He is extremely non-compliant and drug-s eeking, and has slapped one of the nurses in the endoscopy lab for not giving hi m enough pain medications." You have been the patient?s primary care provider f

or the past 12 years and know that the patient can be difficult at times. Which of the following is the most appropriate next step in the management of this pa tient? A. Ask him to withdraw from the patient?s case if he wishes. B. Tell him that he can withdraw from the case but he will not be consulted aga in for any of your patients in the future. C. Ask him to document the patient?s behavior in the chart and then withdraw fr om patient?s case. D. Tell him that he cannot withdraw from the case once he is involved in patien t care. E. Tell him that he can withdraw from the case if some other gastroenterologist is willing to take care of this patient. Answer: E

All physicians have a moral obligation to provide continuity of care for their p atients. It is not appropriate to withdraw or discontinue patient care in the m iddle of a hospitalization or during an acute ongoing medical treatment. All ph ysicians have the option to choose who they want to serve; however, once they ar e involved in a case, they should not neglect the patient. If a physician wishe s to withdraw from a case, he can do so if he provides the patient or the caregi vers a notice long in advance so as to sufficiently permit the transfer of medic al care to another physician. In the above vignette, the gastroenterologist is already involved with the patient's care since he has already performed the proc edure. He can opt to withdraw from the patient?s case only if there is another gastroenterologist willing to assume patient care responsibility. (Choices A and B) Asking the physician to withdraw without securing another gast roenterologist for the patient is not appropriate. (Choice C) The patient?s behavior should certainly be documented in the chart; h owever, this does not provide sufficient grounds for withdrawal in the middle of the treatment. (Choice D) As described above, the physician can withdraw from the case if anoth er gastroenterologist is willing to assume the patient care responsibility. Educational Objective: Physicians can terminate the physician-patient relationship by providing a notic e long in advance to sufficiently permit the transfer of care to another healthc are provider. 29% of people answered this question correctly.

46. A 39-year-old Caucasian man is brought to the emergency department because o f epigastric pain and melena. He was diagnosed of peptic ulcer disease five yea rs ago. He underwent highly selective vagotomy with antral ulcer resection due to persistent gastrointestinal bleeding one year later. He has no other medical problems. He is a real estate agent. He does not use tobacco or illicit drugs , but drinks alcohol occasionally. His medications include omeprazole and antac ids. His blood pressure is 130/80 mm Hg, pulse is 120/min, and respirations are 18/min. He looks pale and frightened. Examination shows a midline scar in the epigastrium. There is moderate tenderness on palpation of the upper half of th

e abdomen. Some muscular guarding can be appreciated. There is no rebound tend erness, and bowel sounds are present. A rectal exam reveals dark blood. During the examination, the patient feels nauseous and has one episode of hematemesis. He receives intravenous hydration and parenteral pantoprazole. Some laborator y tests are done, including cross-typing for a possible transfusion. The result s show the following: CBC Hb: 7.6 g/dL Ht: 23% Platelet count: 450,000/cmm Leukocyte count: 8,000/cmm Segmented neutrophils: 72% Bands: 3% Lymphocytes: 25% Serum chemistry Serum Na: 145 mEq/L Serum K: 3.9 mEq/L Chloride: 110 mEq/L Bicarbonate: 25 mEq/L BUN: 28 mg/dL Serum creatinine: 1.2 mg/dL Calcium: 9.9 mg/dL Glucose: 77 mg/dL Which of the following is the most appropriate test to confirm the diagnosis? A. Abdominal ultrasound B. Upper gastrointestinal (GI) radiology series C. Abdominal computerized tomography (CT) with contrast D. Upper gastrointestinal (GI) endoscopy E. Radionuclide imaging Answer: D

There is a significant rate of rebleeding after gastric surgery for peptic ulcer treatment. For instance, truncal vagotomy with partial antrectomy has a recurr ence rate of 5 % to 10% after a mean follow-up of 3.5 years. The preferred meth od to confirm the source of bleeding is upper gastrointestinal (GI) endoscopy, b ecause it also has therapeutic applications such as photocoagulation or local in jection of vasoconstrictor agents. In addition, early endoscopy has been associ ated with a significant decrease in mortality and hospitalization time. (Choice A) Abdominal ultrasound is not adequate for the evaluation of gastrointe stinal bleeding because it cannot identify intraluminal lesions. (Choice C) Abdominal CT scan is not regularly used to evaluate GI bleeding becau se endoscopy is superior to it. Abdominal CT scan with contrast can sometimes l ocalize the source of bleeding of lower gastrointestinal hemorrhages. (Choice B) Although a contrast study of the esophagus and the stomach can certai nly show the presence of ulcerated lesions, it does not offer therapeutic advant ages. This is why it is not recommended for a patient who needs immediate evalu ation, such as those who are actively bleeding. (Choice E) Radionuclide scanning is less specific than upper GI endoscopy. Its accuracy depends on the experience of the operator and the type of clinical cent er; values may be as low as 24% or as high as 91%. Educational Objective:

Upper GI endoscopy is the preferred method to evaluate upper gastrointestinal bl eeding because it also has therapeutic applications. The most common reason for bleeding in a patient with a history of previous gastric lesion is the developm ent of a new ulcerated lesion or the recurrence of the previous one. These lesi ons are readily diagnosed and managed through endoscopy. 84% of people answered this question correctly.

47. Two groups of investigators are interested in the problem of esophageal canc er in a population that has a high prevalence of hot beverage consumption. One group conducted study A, which showed that hot beverage consumption is associate d with esophageal cancer, with an odds ratio (OR) of 1.51 and 95% confidence int erval of 1.19 to 1.81. The other group conducted study B, which demonstrated an OR of 1.46 and 95% confidence interval of 0.95 to 1.97. Which of the followin g is the best statement concerning the results of these two studies? A. B. C. D. E. The result The sample The result The sample P value in D of study A is not statistically significant in study B is poorly selected in study A is not valid size in study B is small study B is less than 0.05

Answer:

Although both studies have almost the same point estimate of association (OR of about 1.5), study B has a wider confidence interval than study A. Furthermore, the confidence interval of study B includes 1.0; therefore, it is not statistica lly significant. The lack of statistical significance of the results in study B is most probably due to a smaller sample size, which resulted in insufficient p ower to detect the difference between the exposed and unexposed subjects. Incre asing the sample size of study B will make the confidence interval tighter. Bot h studies reached the same conclusion, but because of a smaller sample size of s tudy B, its estimated odds ratio does not reach statistical significance. (Choice A) The 95% confidence interval in study A does not include the ?null? va lue for the odds ratio (that is 1.0); therefore, the odds ratio achieves statist ical significance. (Choices B and C) The accuracy or validity of the studies cannot be evaluated wi thout any information on how the studies were designed and conducted. (Choice E) The P value of statistically insignificant results should be greater than 0.05. Educational Objective: The power of a study is the ability to detect the difference between two groups (treated vs non-treated, exposed vs non-exposed). Increasing the sample size in creases the power of a study. As a result, the confidence interval of the point estimate (e.g., odds ratio) becomes tighter. 25% of people answered this question correctly.

48. A 58-year-old female with a history of diabetes mellitus presents to the fam ily physician for diarrhea, weight loss, bloating, and excess flatulence. She w

as diagnosed with diabetic enteropathy last year. Laboratory examinations show anemia with MCV of 101 fl. Which of the following is the best test to diagnose bacterial overgrowth in this patient? A. B. C. D. E. Quantitative jejunal cultures [14C]-d-xylose breath test Bentiromide test Breath hydrogen analysis Quantitative fecal fat A

Answer:

The demonstration of excessive bacterial concentrations in a jejunal aspirate is the gold standard for the diagnosis of bacterial overgrowth. It can be perform ed during endoscopy or by fluoroscopy with jejunal intubation. The patient's ba cterial overgrowth is most likely due to her diabetic enteropathy. (Choice B) The Clinical Efficacy Committee of the American College of Physicians also advocates alternate use of one-gram [14C]-d-xylose in routine clinical pra ctice. Xylose is a pentose sugar that is catabolized by gram-negative aerobes p resent in the intestine, releasing the radioactive isotope 14-CO2, which is dete ctable in breath samples. (Choice D) Breath hydrogen testing is performed by the administration of a test dose of carbohydrate, which would be associated with a rise in breath hydrogen l evels in patients with bacterial overgrowth. (Choice C) A bentiromide test is a test for pancreatic insufficiency and not bac terial overgrowth. (Choice E) Quantitative fecal fat is a test for fat malabsorption, but is not sp ecific for bacterial overgrowth. Educational Objective: The gold standard for the diagnosis of bacterial overgrowth is the demonstration of excessive bacterial concentrations in a jejunal aspirate. 25% of people answered this question correctly.

49. You are managing the care of a 32-year-old football player in the intensive care unit. He was involved in a motor vehicle accident two days ago, and had su stained severe head injuries. The paramedics intubated him at the scene of the accident. On the third day of hospitalization, he remains unresponsive to all s timuli and has no spontaneous respiratory drive. He is on full ventilator suppo rt. His temperature is 36.7 C (98 F), blood pressure is 120/70 mmHg, and pulse is 76/min. He is declared braindead by the ICU team. His family is informed of his status, and they wish to proceed with the withdrawal of life support. His fianc?nforms you that he had always wanted to donate his organs in case of an un expected death. Which of the following is important to maximize the viability o f donor organs? A. Anti-hypertensive medications to keep systolic blood pressure less than 100 mmHg B. Low-dose beta blockers to keep heart rate less than 60/minute C. Lower the ventilator rate to induce hypercapnia

D. Maintain body temperature to less than 35 C (95 F) E. Maintain normothermia with blankets Answer: D

Organ transplantation from cadaveric donors has become increasingly common in th e last few years. Most of the organs are obtained from brain dead donors. The successful recovery of viable organs for transplantation depends on the appropri ate medical care of brain dead patients; therefore, it is important to learn the basic principles of management of these patients. The primary aim is to achiev e hemodynamic stability and to maintain physiologic homeostasis to maximize the viability of organs. A common misconception is that the body temperature should be kept low to improve the viability of organs. In contrast, normothermia shou ld be maintained passively in all brain dead patients using blankets. If the bo dy temperature is less than 35 C (95 F), active rewarming should be attempted wi th warm air blankets and warm intravenous fluids. (Choices A and B) It is critical to achieve and maintain a normotensive, euvolem ic state. Hypotension is very common in brain dead patients, and may be due to the loss of sympathetic tone, systemic infections, volume depletion secondary to losses, and diabetes insipidus. Adequate volume resuscitation is the first ste p in correcting hypotension and ensuring donor viability. If systemic blood pre ssure still remains low, pressors and inotropic agents may be used to improve th e blood pressure. (Choice C) Hypercapnia has not been shown to improve organ viability, and can be even damaging to the tissues and organs. The goal of management is to provide optimal ventilator support to prevent hypoxia and hypercapnia. Educational Objective: The primary goal of medical management of a brain dead organ donor is to maintai n a euvolemic, normotensive, and normothermic state. 45% of people answered this question correctly. 50. A 30-year-old male is brought to the emergency department (ED) by his relati ves following a generalized tonic-clonic seizure. Physical examination shows te tany and cramping of muscles. Chvostek and Trousseau signs are positive. The E KG tracing shows prolongation of QT intervals. His blood pressure is 100/70 mmH g, pulse is 67/min, temperature is 36.7C (98F), and respirations are 12/min. Hy pocalcemia is suspected. Apart from measuring the patient's serum calcium level s, which of the following is the most important laboratory investigation to cond uct? A. B. C. D. E. Serum parathyroid levels Serum phosphorus levels Serum albumin levels 25-hydroxy vitamin D levels Serum glucose levels E

Answer:

This patient has clinical features suggestive of hypocalcemia. Aside from measu rement of the serum calcium level, measurement of the serum glucose level is als o very important. Hypoglycemia is the greatest mimmicker of central nervous sys tem diseases. It can present with protean signs and symptoms. One remote possi bility of the association of hypocalcemia and hypoglycemia is autoimmune polygla ndular failure, where hypoparathyroidism (decreased calcium levels) is associate d with primary adrenocortical insufficiency (decreased glucose levels).

(Choices A, B, C & D) Measurement of the serum parathyroid hormone, phosphorous, albumin and vitamin D levels are also required in evaluation of patients with h ypocalcemia; however, measurement of the serum glucose levels should be done rig ht away by doing a fingerstick. The PTH level is low in patients with hypoparat hyroidism, and increased in patients with pseudohypoparathyroidism and vitamin D deficiency. The serum phosphorus level is increased in patients with hypoparat hyroidism and low in vitamin D deficiency. Approximately 50% of calcium is boun d to albumin; therefore a change in the serum albumin levels can result in a cha nge of the total calcium levels without a change in the ionized fraction. Total serum calcium levels should always be corrected for corresponding serum albumin level. Hypovitaminosis D is a very important cause of hypocalcemia and can be assessed by measuring the 25-hydroxy vitamin D levels. Educational Objective: Hypoglycemia can lead to protean central nervous system manifestations which inc lude focal neurological deficits, alerted mental status, seizures, and coma. Se rum glucose levels should therefore be measured in all patients presenting with central nervous system dysfunction. 51. A healthy 14-year-old Caucasian girl comes to your office and asks for a pre scription for oral contraceptive pills. She recently started having sex with he r boyfriend, who is 16 years old. She started menstruating at the age of 13, an d she has always had regular cycles. She is a non-smoker and non-alcoholic. Sh e does not have any medical problems. She uses condoms infrequently. Which of following is the most appropriate next step in the management of this patient? A. B. C. D. E. Prescribe her oral contraceptives and inform the parents later. Call the parents from the office and prescribe if they are okay. Advise her to use barrier methods instead of oral contraceptives. Advise her to use barrier methods in addition to oral contraceptives. Educate about teenage sex and do not give contraception. D

Answer:

This patient does not have any absolute or relative contraindication to the use of oral contraceptive pills (OCPs) and has a personal preference for this mode o f contraception; therefore, she should be prescribed OCPs. In addition, barrier methods should be offered to reduce her risk for STDs. (Choices A and B) Adolescents do not need consent from their parents for the use of oral contraceptives. The biggest reason for failure of adolescents to obtai n contraceptive services is the concern about confidentiality; therefore, confid entiality should always be maintained. Educational Objective: Adolescents do not need consent from their parents for contraceptive services. Barrier methods should be used by all sexually active adolescents even if an add itional method of contraception is being used.

52. A 16-year-old Caucasian girl is being worked up for primary amenorrhea. She has short stature, webbed neck, high palate, and a short fourth metacarpal. He r karyotype analysis showed 45, XO. Her mother is concerned about the patient's chances to conceive. Which of the following is the best response? A. B. C. D. E. IV labetalol She has the same chances of pregnancy as the general population. She can become pregnant, but pregnancy is contraindicated in her condition. Pregnancy is absolutely impossible. Educate about teenage sex and do not give contraception. D

Answer:

Ovarian function and morphology is highly variable in patients with Turner syndr ome. Most patients do not undergo puberty, and present with primary amenorrhea. Some patients may develop normally and undergo spontaneous menarche, but secon dary amenorrhea may occur after some time. Such patients may become pregnant wi thout medical assistance before developing secondary amenorrhea; however, most w omen are infertile. In one retrospective study of 522 patients, three women bec ame pregnant spontaneously. (Choice C) Turner syndrome is not a contraindication for pregnancy; however, pat ients should undergo a complete medical evaluation. Educational Objective: Pregnancy is almost impossible, but there still remains a very small chance, in patients with Turner syndrome. 42% of people answered this question correctly. The following vignette applies to the next 2 items

53. A 65-year-old Caucasian male presents to the emergency department six hours after the onset of left-sided weakness in his arm and leg. He denies headache, nausea, vomiting and swallowing difficulty. He had two episodes of extremity we akness and numbness within the last three months that lasted 15-20 minutes and r esolved spontaneously. His past medical history is significant for hypertension and diabetes. His current medications are hydrochlorothiazide, lisinopril and metformin. He also takes ibuprofen for muscle cramps and antacids for heartburn s occasionally. He does not smoke or consume alcohol. His family history is in significant. His blood pressure is 170/100 mmHg and heart rate is 80/min, regul ar. The CT scan of his head is negative for intracranial hemorrhage.

What is the best next step in the management of this patient? A. IV labetalol B. IV isotonic D5W and oral metoprolol C. IV isotonic saline and aspirin D. IV isotonic saline and nimodipine E. Lumbar puncture Answer: C

The acute management of ischemic stroke emphasizes the concept of ?first, do no harm.? Despite very intensive clinical research in this field, very few interve ntions have been proved to be beneficial. Cautious IV hydration with isotonic s aline is generally indicated. Aspirin has been shown to be effective in improvi ng prognosis in two large-scale clinical trials, and is recommended in all patie nts with ischemic stroke who do not receive thrombolytics or anticoagulants. (Choice A) Some interventions that sometimes seem intuitively appropriate can be harmful. For example, the reduction of blood pressure in patients with acute i schemic stroke can deteriorate their condition, and should be used only if the b lood pressure is extremely high (>220/120). If blood pressure reduction is requ ired, labetalol is the drug of choice. (Choice B) Overzealous hydration and D5W solution are usually avoided. (Choice D) Despite initial encouraging evidence, large-scale studies demonstrate d no improvement in the outcome of patients with thrombotic stroke receiving nim odipine. (Choice E) Lumbar puncture is indicated when a patient has a negative CT scan an d the clinical suspicion of subarachnoid hemorrhage remains high. Educational Objective: The reduction of blood pressure in patients with acute ischemic stroke can deter iorate their condition, and should be used only if the blood pressure is extreme ly high (>220/120). Despite initial encouraging evidence, large-scale studies d emonstrated no improvement in the outcome of patients with thrombotic stroke rec eiving nimodipine. 48% of people answered this question correctly. 54. A 65-year-old Caucasian male presents to the emergency department six hours after the onset of left-sided weakness in his arm and leg. He denies headache, nausea, vomiting and swallowing difficulty. He had two episodes of extremity we akness and numbness within the last three months that lasted 15-20 minutes and r esolved spontaneously. His past medical history is significant for hypertension and diabetes. His current medications are hydrochlorothiazide, lisinopril and metformin. He also takes ibuprofen for muscle cramps and antacids for heartburn s occasionally. He does not smoke or consume alcohol. His family history is in significant. His blood pressure is 170/100 mmHg and heart rate is 80/min, regul ar. The CT scan of his head is negative for intracranial hemorrhage The patient receives the appropriate therapy. Two days after the event, he does not show any progression of neurological deficit. The EKG shows no rhythm abno rmalities, and carotid duplex scanning is normal. What is the best next step in the management of this patient? A. Discharge the patient B. Order echocardiography C. Repeat CT scan D. Schedule carotid angiography E. Obtain EEG

Answer:

Determining the stroke subtype is important in patients with ischemic stroke in terms of short-term as well as long-term treatment decisions. Thrombotic, embol ic and lacunar strokes may require different management approaches that undersco re the importance of the underlying pathophysiology. Cardiac evaluation should be performed in almost all patients with brain ischemia. Echocardiography is in dicated in patients with suspected embolic stroke and in patients in whom the me chanism of stroke is not clear, especially if carotid studies are negative. Lef t ventricular systolic dysfunction appears to be more common in patients who pre sent with stroke than in the general population, and many patients with stroke h ave asymptomatic left ventricular dysfunction that can be detected on echocardio graphy. (Choice C) In stable patients with no progression of neurological deficits, repe at CT scan has little diagnostic significance. (Choice D) Conventional angiography is rarely performed nowadays. It may be hel pful in patients with fibromuscular dysplasia and carotid dissection, as well as patients undergoing preoperative evaluation of carotid artery disease. Educational Objective: Echocardiography is indicated in patients with suspected embolic stroke and in p atients in whom the mechanism of stroke is not clear, especially if carotid stud ies are negative. 42% of people answered this question correctly. 55. A 55-year-old Caucasian female is brought to the emergency department by her son because of slurred speech and confusion. These symptoms started several da ys ago and gradually progressed. Her past medical history is significant for hy pertension, bipolar disorder, and deep vein thrombosis. Her son says that the ? mood disorder? runs in the family because his grandmother had the same problem. She takes lithium, which seems to control her symptoms well. One month ago, sh e was diagnosed with deep vein thrombosis and hospitalized. Warfarin treatment was started at the hospital, and she continues to take this drug with periodic ? blood tests?. The physical examination reveals a mild tremor of her hands and a n unstable gait. She is oriented to person, but not to time and space. If medi cation toxicity is responsible for this patient?s current symptoms, which of the following is the most likely additional finding? A. B. C. D. E. Hormone replacement therapy (HRT) for hot flushes Hydrochlorothiazide for hypertension treatment Low INR value on routine coagulation studies Right-sided hemiparesis with slight motor aphasia Bilateral hemianopsia on visual field testing B

Answer:

Lithium is widely used as an effective agent to treat bipolar disorder. It has a low therapeutic index. An overdose may result due to intentional poisoning or other factors. The mortality is 25% in patients with acute lithium poisoning a nd 8% in patients on maintenance dose. Manifestations of lithium toxicity are d iverse: gastrointestinal, renal, endocrine, dermal, cardiovascular, hematologica l, as well as neurological abnormalities have been described. A significant num ber of patients suffers irreversible neurological damage. Typical neurological symptoms include lethargy, confusion, tremors, fasciculations and ataxia. This scenario demonstrates how various factors can affect serum lithium levels and pr ecipitate lithium toxicity. Any factor that decreases renal excretion, such as

renal insufficiency or effective volume depletion (e.g., diuretics or congestive heart failure) may be responsible. (Choice A) HRT may be responsible for deep vein thrombosis in this patient, but the question asked about the current neurological symptoms. (Choice C) A low INR value on routine coagulation studies indicates inadequate a nti-coagulation. (Choices D and E) Right-sided hemiparesis would suggest a stroke, and bilateral hemianopsia on visual field testing would most probably indicate the presence of an intracranial mass lesion in this patient. Educational Objective: Any factor that decreases renal excretion may be responsible for the increase of serum lithium levels. 25% of people answered this question correctly.

56. A 61-year-old male with hypercholesterolemia, diabetes mellitus type 2 and h ypertension comes to the office because of constipation. He has hyperlipidemia, hypertension and diabetes mellitus controlled with atorvastatin, hydrochlorothi azide and rosiglitazone, respectively. He denies alcohol intake, but admits smo king two packs of cigarettes daily for the last several years. Colonoscopy perf ormed one year ago was normal. Lab investigations reveal a serum calcium level of 14.2 mg/dL and an albumin level of 4.0 gm/dL. What is the most likely cause of this patient?s hypercalcemia? A. Primary hyperparathyroidism B. Secondary hyperparathyroidism C. Drug-induced D. Malignancy E. Diabetic nephropathy Answer: D

This patient has a longstanding history of smoking; therefore, he has higher cha nces of developing lung cancer. Patients with squamous cell carcinoma of the lun g can produce parathyroid hormone related protein (PTHrP), which can cause hyper calcemia. It is sometimes difficult to differentiate primary hyperparathyroidis m from hypercalcemia seen during malignancy. Typically, the serum calcium level in patients with malignant lung tumors is much higher than those with primary h yperparathyroidism. This patient?s serum calcium level is 14.2 mg/dL, and is mo st likely due to a malignancy. (Choice A) Primary hyperparathyroidism is one of the most common causes of hyper calcemia in an outpatient setting. As stated above, the level of serum calcium is lower in patients with primary hyperparathyroidism compared to patients with malignancy-induced hypercalcemia. Calcium levels above 12mg/dL are rare. (Choice B) Secondary hyperparathyroidism due to renal failure or vitamin D defic iency is not typically associated with hypercalcemia. Patients with secondary h yperparathyroidism can sometimes develop autonomously functioning nodules of the parathyroid gland, which is then called tertiary hyperparathyroidism. Hypercal

cemia can occur in the presence of very high serum PTH levels in tertiary hyperp arathyroidism. (Choice C) HCTZ can cause hypercalcemia, but the elevations in serum calcium lev els are usually minimal. (Choice E) Diabetic nephropathy per se is not associated with hypercalcemia. Educational objectives: The three most common causes of hypercalcemia include primary hyperparathyroidis m, malignancy-induced hypercalcemia and vitamin D-induced hypercalcemia. In hyp ercalcemia secondary to malignancy, the serum calcium levels are generally very high. 34% of people answered this question correctly.

57. A 61-year-old male with hypercholesterolemia, diabetes mellitus type 2 and h ypertension comes to the office because of constipation. He has hyperlipidemia, hypertension and diabetes mellitus controlled with atorvastatin, hydrochlorothi azide and rosiglitazone, respectively. He denies alcohol intake, but admits smo king two packs of cigarettes daily for the last several years. Colonoscopy perf ormed one year ago was normal. Lab investigations reveal a serum calcium level of 14.2 mg/dL and an albumin level of 4.0 gm/dL The physical examination of the patient is essentially unremarkable. What is th e most appropriate next step in management? A. Serum PTH measurement B. Bone scan C. CT scan of the chest D. Chest x-ray E. Renal biopsy Answer: A

Hypercalcemia can be divided in to two broad categories: (1) parathyroid depende nt and (2) parathyroid independent. Examples of parathyroid dependent hypercalc emia include primary or tertiary hyperparathyroidism, lithium-induced hypercalce mia, and familial hypocalciuric hypercalcemia. Parathyroid independent causes i nclude malignancies, vitamin D toxicity, granulomatous disease, and milk-alkali syndrome. Malignancy-induced hypercalcemia is generally due to the secretion of PTHrP, but other mechanisms have also been described, particularly in hematolog ical malignancies. These categories can be distinguished by measuring the serum parathyroid hormone levels; therefore, the most appropriate next step in this case is to do a parat hyroid estimation to help classify this patient's hypercalcemia as parathyroid d ependent or independent. Specific PTHrP assays are commercially available and a re usually ordered with PTH if malignancy is highly being considered in the diff erential diagnosis. (Choices B, C, D and E) Bone scan, CT scan, chest x-ray, and renal biopsy are no

t indicated at this point. In majority of endocrinological disorders, biochemic al confirmation is required before any imaging is performed. Educational Objective: Hypercalcemia can be divided in to two broad categories: (1) parathyroid depende nt and (2) parathyroid independent. The most appropriate next step in managemen t is to do a parathyroid estimation.

58. A 36-year-old African-American man was brought to the hospital because of fe ver, chills, productive cough, and shortness of breath. He also has night sweat s, loss of appetite, and weight loss. He is unemployed and homeless; he was fou nd in the street and brought to the hospital by some concerned policemen. He ha s no other medical problems. He drinks 6-8 bottles of beer daily. His family history is not significant. He has no medications. He has no known drug allerg ies. His temperature is 38.9 C (102 F), pulse is 104/min, and respirations are 22/min. His pulse oximetry is 92% at room air. His chest x-rays show a left up per lobe alveolar infiltrate. He is admitted and started on clindamycin and gat ifloxacin. On his fourth hospital day, he is breathing better and has more appe tite, but he continues to be febrile. His laboratory results reveal that one of his sputum samples is positive for acid-fast bacilli (AFB), and he is started o n anti-tuberculosis (TB) therapy. The nurses that were taking care of him in th e unit approach you to ask about their risk of acquiring tuberculosis (TB). The y want you to tell them what they can do about it. Which of the following is th e most appropriate course of action? A. Place a PPD in three weeks and evaluate B. Place a PPD now and repeat it in three weeks C. Take a chest x-ray in three weeks D. Start prophylactic therapy with isoniazid (INH) for those who attended him f or more than two consecutive days without using a mask E. Take a chest x-ray and place a PPD in three weeks Answer: B

The Center for Disease Control (CDC) recommends immediately giving a PPD test to all health care workers (HCW) exposed to a contagious patient with tuberculosis . If the result is negative, the PPD test will be repeated after three weeks to check for any changes that could have been induced by the recent exposure to th e bacteria. (Choice D) Prophylactic therapy with INH is not needed at this point, as it is n ot known if the HCWs were infected or not. (Choices C and E) PPD test must be given before ordering a chest x-ray, unless t he HCW has specific respiratory symptoms. (Choice A) The initial PPD must always be given immediately because the HCW can be PPD positive from before, and not due to the most recent exposure. Educational Objective: When a HCW is exposed to a patient with a contagious form of tuberculosis (i.e., laryngeal, bronchial, or pulmonary), the CDC recommends immediate PPD testing t o determine the baseline immunologic status; three weeks after, PPD retesting wi ll be given to check for any changes due to the recent TB exposure. 59. A 25-year-old Caucasian man comes to the emergency department because of per

sistent pain and limited range in motion of the right wrist. Five hours ago, wh ile playing football, he fell to the ground and landed on his outstretched right hand. He went home after the incident because his hand did not look too swolle n and the pain was tolerable. A few hours later, he noticed that he was unable to move his hand, and decided to come to the hospital. He has no other medical problems. He denies the use of tobacco, alcohol or drugs. Examination shows mi ld swelling in the dorsum of the right hand. The hand cannot be fully flexed, a nd can be extended only by passive motion due to pain. There is severe tenderne ss on palpation of the anatomical snuffbox. Which of the following is the most likely diagnosis? A. Colles? fracture B. Chondral fracture C. Wrist sprain D. Scaphoid fracture E. Triangular cartilage injury Answer: D

A history of falling on an outstretched hand is characteristic of a scaphoid fra cture. It is most commonly seen in young adults between the ages of 15 and 30 y ears. It results from a fall on the outstretched hand, resulting in severe hype rextension and slight radial deviation of the wrist. Accompanying symptoms are complete (or greater than 50%) loss in range of motion of the wrist joint, sever e pain, and stiffness. Physical examination reveals tenderness on palpation of the scaphoid within the anatomic snuffbox. (Choice C) Wrist sprain is characterized by mild pain, stiffness, normal range o f motion of the wrist joint, minimal tenderness on palpation of the lunate and s caphoid (navicular) bones, and mild swelling of the dorsum. (Choice B) Chondral fracture is characterized by moderate pain, stiffness, and a pproximately 20% loss in range of motion of the wrist joint. (Choice E) Triangular cartilage injury is characterized by tenderness of the reg ion distal to the ulnar styloid, which increases in severity with forced ulnar d eviation. (Choice A) Patients with Colles? fracture have tenderness in the region located 2 cm below the radioulnar joint. In almost half of the cases, the ulnar styloid separates from the rest of the bone, and lateral view of the wrist reveals a di nner fork deformity. Educational Objective: A history of falling on an outstretched hand, complete (or greater than 50%) los s in range of motion of the wrist joint, severe pain, and tenderness on palpatio n of the scaphoid within the anatomic snuffbox is diagnostic of scaphoid fractur e until proven otherwise. 60. A 25-year-old Caucasian man comes to the emergency department because of per sistent pain and limited range in motion of the right wrist. Five hours ago, wh ile playing football, he fell to the ground and landed on his outstretched right hand. He went home after the incident because his hand did not look too swolle n and the pain was tolerable. A few hours later, he noticed that he was unable to move his hand, and decided to come to the hospital. He has no other medical problems. He denies the use of tobacco, alcohol or drugs. Examination shows mi ld swelling in the dorsum of the right hand. The hand cannot be fully flexed, a nd can be extended only by passive motion due to pain. There is severe tenderne ss on palpation of the anatomical snuffbox.

An x-ray of the wrist reveals no evidence of dislocation or fracture. After rec eiving analgesic therapy, the patient reports that the pain in his wrist has imp roved. Which of the following is the most appropriate next step in the manageme nt of this patient? A. Prescribe oral analgesics, rest for two weeks, and discharge the patient hom e B. Order a computerized tomography (CT) scan of the right wrist C. Order a magnetic resonance imaging (MRI) study of the right wrist D. Place a cast brace with the hand on dorsal hyperextension E. Place a cast brace with the hand on dorsal flexion Answer: B

Patients with a non-displaced scaphoid fracture can have normal radiographs for up to two weeks after a traumatic incident. Overlooking this lesion may lead to complications such as traumatic arthritis (which results from nonunion of the f racture) and avascular necrosis of the scaphoid bone. A high clinical suspicion for scaphoid fracture warrants the use of further diagnostic studies (i.e., CT scan of the hand, bone scan), even if the initial x-ray results are negative. (Choice A) Analgesic therapy and resting the affected hand are indicated in the management of wrist sprain. (Choice C) MRI studies are used in the diagnosis of triangular cartilage injury. Although it may also be used in this case, its higher cost makes it less appea ling than a CT scan, which is enough to make the diagnosis. (Choice E) Cast placement in the management of a patient with uncomplicated Coll es? fracture involves immobilization of the hand in the neutral position, or ide ally, in the normal volar tilt position. (Choice D) Cast placement with the hand in a hyperextended position is not benef icial for both Colles? and scaphoid fractures. The uncomplicated, undisplaced s caphoid fractures are treated with immobilization in a long or short arm thumb s pica cast with the wrist in slight radial deviation and in neutral flexion.

Educational Objective: Although displaced fractures of the scaphoid can be identified in x-rays immedia tely after trauma, some scaphoid fractures can take one or two weeks before it b ecomes apparent in x-rays. A high clinical suspicion for scaphoid fracture ther efore warrants the use of further diagnostic studies (i.e., CT scan of the hand, bone scan), even if the initial x-ray results are negative. The uncomplicated, undisplaced scaphoid fractures are treated with immobilization in a long or sho rt arm thumb spica cast with the wrist in slight radial deviation and in neutral flexion. 9% of people answered this question correctly. 61. A 25-year-old Caucasian man comes to the emergency department because of per sistent pain and limited range in motion of the right wrist. Five hours ago, wh ile playing football, he fell to the ground and landed on his outstretched right hand. He went home after the incident because his hand did not look too swolle n and the pain was tolerable. A few hours later, he noticed that he was unable to move his hand, and decided to come to the hospital. He has no other medical problems. He denies the use of tobacco, alcohol or drugs. Examination shows mi ld swelling in the dorsum of the right hand. The hand cannot be fully flexed, a nd can be extended only by passive motion due to pain. There is severe tenderne ss on palpation of the anatomical snuffbox.

The patient is concerned about the complications that he might develop. Which o f the following is the most common complication of his injury? A. Avascular necrosis B. Nonunion C. Malunion D. Infection E. No complications Answer: B

The most common complication of scaphoid fractures is nonunion. The other impor tant complication is avascular necrosis. These two complications often result b ecause the blood flows from the distal to proximal portion of the scaphoid bone, and this proximal portion is completely dependent on the distal blood supply. Proximal fractures of the scaphoid therefore require longer immobilization (up t o 12 weeks) to ensure adequate healing. Educational Objective: The most common complication of scaphoid fracture is nonunion. 9% of people answered this question correctly.

62. A follow-up study was conducted to assess the effect of circumcision on the incidence of penile carcinoma. A large cohort was investigated. The rate in un circumcised males was 8/1000, and the rate in circumcised males was 6/1000. The investigators concluded that circumcision may be markedly effective in reducing the incidence of the disease in the population. According to the study results , how many patients should be circumcised to prevent one case of penile carcinom a? A. B. C. D. E. 100 250 500 1000 5000 C

Answer:

The "number needed to treat" (NNT) is the number of patients who need to be trea ted in order to prevent one additional bad outcome. It is an important way to p resent the results of a study or assess the usefulness of treatment or prophylax is. Sometimes, it is more convenient for practitioners to use NNT than measures of association (which represents the strength of association, not the practical aspects of treatment efficacy). The calculation of NNT is easy: it is actually the inverse of absolute risk reduction (ARR). ARR = Control group event rate ? Experimental group event rate = 8/1000 ? 6/1000 = 2/1000 NNT = 1/ARR = 1/0.002 = 500

Therefore, according to the study results, 500 males should be circumcised in or der to prevent one case of penile carcinoma. Educational Objective: NNT is the number of patients who need to be treated in order to prevent one add itional bad outcome; it is actually the inverse of absolute risk reduction.

63. A 21-year-old Caucasian male presents to your office with a non-pruritic ras h. He says that he noticed it in the spring season, and it got worse during the summer. His past medical history is insignificant. He does not smoke or consu me alcohol, and denies any recreational drug use. He has been sexually active w ith two partners during the last year, and he uses condoms occasionally. The ph ysical examination reveals multiple circular hypopigmented macules on the chest, back and upper arms. Item 1 of 2 Which of the following is the most likely diagnosis in this patient? A. Pityriasis rosea B. Tinea versicolor C. Secondary syphilis D. Eczema E. Rocky mountain spotted fever Answer: B

Tinea versicolor is a fungal infection of the skin that is caused by the dimorph ic yeast Pityrosporum orbiculare (also known as Malassezia Furfur). The clinica l picture is usually very characteristic: multiple small circular maculae are ob served that may vary in color (white, pink or brown). The rash is typically mor e prominent in the summer time because the yeast inhibits pigment transfer to ke ratinocytes and makes the affected skin more demarcated from unaffected tanned s kin. The lesions are usually asymptomatic, although mild pruritus may be presen t. The typical location of the lesions is the upper trunk, but the rash may als o involve the upper arms, neck and abdomen. (Choice A) Pityriasis rosea is characterized by the presence of a herald patch, confinement of the lesions to the central trunk, orientation of the lesion along the lines of cleavage of the skin, and a typical pink color of the lesions. (Choice C) Secondary syphilis commonly involves the hands and feet. (Choice D) Eczema typically affects the extremities and produces scaling and sev ere pruritus. (Choice E) RMSF is an infectious disease that is characterized by an acute onset and severe systemic symptoms.

Educational Objective: In patients with tinea versicolor, multiple small circular maculae are observed which may vary in color (white, pink or brown). 75% of people answered this question correctly. 64. A 21-year-old Caucasian male presents to your office with a non-pruritic ras h. He says that he noticed it in the spring season, and it got worse during the summer. His past medical history is insignificant. He does not smoke or consu me alcohol, and denies any recreational drug use. He has been sexually active w ith two partners during the last year, and he uses condoms occasionally. The ph ysical examination reveals multiple circular hypopigmented macules on the chest, back and upper arms. Which of the following is the best treatment for this patient? A. Oral erythromycin B. Topical terbinafine C. Penicillin IM D. Topical corticosteroids E. Oral doxycycline Answer: B

The treatment of choice for patients with tinea versicolor is topical anti-funga l therapy. Any anti-yeast topical agent can be used, including terbinafine, clo trimazole and ketoconazole. The success rate with topical anti-fungal agents ex ceeds 80%. With extensive disease or recalcitrant infection, oral antifungals a re preferred (ketoconazole, itraconazole or fluconazole). (Choice A) Oral erythromycin is effective in patients with pityriasis rosea. (Choices C) Penicillin is used for the treatment of syphilis. (Choice E) Doxycycline is used in the treatment of patients with RMSF. (Choice D) Topical corticosteroids can be used in patients with eczema. Educational Objective: The treatment of choice for patients with tinea versicolor is topical anti-funga l therapy. 70% of people answered this question correctly.

65. A 65-year-old Caucasian female presents to the emergency department with pro gressive shortness of breath on minimal exertion and fatigue. She was diagnosed with myelodysplasia two years ago, and has been receiving supportive therapy wi th frequent RBC transfusions. Her last transfusion was 3 months ago. She exper ienced an episode of severe pneumonia one year ago that required hospitalization and IV antibiotic therapy. She is taking no medications currently and has no k nown allergies. Her blood pressure is 120/70 mmHg and heart rate is 95/min. Ph

ysical examination reveals pallor. Systolic murmur with intensity of II/VI is h eard over the cardiac apex. Laboratory findings are significant for hematocrit of 24% and hemoglobin level of 5.7 mg/dL. You consider RBC transfusion in this patient. Blood grouping and cross-matching are done, but the blood bank is unab le to find suitable blood. This is the first time such an incompatibility has o ccurred. What is the most likely reason for this incompatibility? A. B. C. D. E. Autoantibodies Alloantibodies Anti-Rh (D) antibodies Anti-HLA antibodies ABO incompatibility B

Answer:

After blood is ordered for transfusion, the following compatibility testing is u sually performed. First, the patient?s ABO and Rh types are determined. After this, the patient's serum is screened for unexpected antibodies, a procedure cal led pretransfusion antibody screening. Pretransfusion antibody screening is int ended to detect any of all clinically significant RBC antibodies. If negative, the patient can be safely transfused. If positive, further investigation is usu ally warranted to evaluate the identity of the antibody. The major problem that leads to difficulties finding cross-matched blood in patients with a history of multiple transfusions is alloantibodies (e.g., in patients with sickle cell ane mia or myelodysplasia). The most commonly implicated RBC antigens in that case are E, L and K. Moreover, these patients tend to develop multiple alloantibodie s that make finding compatible blood even more difficult. (Choices C and E) Finding ABO and Rh-compatible blood is usually not a big chall enge. (Choice A) Autoantibodies are less likely to cause difficulties in cross-matchin g in this patient; they are commonly implicated as a cause of the incompatible c ross-match in patients with autoimmune anemia and taking certain drugs (e.g., me thyldopa and procainamide). (Choice D) HLA allosensitization increases risk of graft rejection in patients a waiting organ or bone marrow transplantation and platelet refractoriness in thos e requiring subsequent platelet transfusion support. Remember that RBCs do not express HLA antigens. Educational Objective: The major problem that leads to difficulties finding cross-matched blood in pati ents with a history of multiple transfusions is alloantibodies. 52% of people answered this question correctly.

You might also like